You are on page 1of 80

Số 03

2019

TẬP SAN TOÁN HỌC


Hướng tới kỳ thi HSG Quốc gia

Nguyễn Tăng Vũ - Nguyễn Ngọc Duy - Vương Trung Dũng


Lê Phúc Lữ - Trần Bá Đạt
Mục lục

1 Công thức nội suy Lagrange 3

2 Một số kĩ thuật giải phương trình hàm trên tập R+ 16

3 Định lý Wolstenholme và ứng dụng 33

4 Bổ đề ERIQ và ứng dụng 43

5 Đếm bằng hai cách trong tổ hợp 54

6 Đề thi tham khảo hướng tới VMO 2019-2020 63

1
2

NGUYỄN TĂNG VŨ - NGUYỄN NGỌC DUY - VƯƠNG TRUNG DŨNG


LÊ PHÚC LỮ - TRẦN BÁ ĐẠT

TẬP SAN TOÁN HỌC


STAR EDUCATION
Số thứ 03 - 2019

Tập san Toán học STAR EDUCATION vừa qua đã ra mắt được hai số và được đông
đảo bạn đọc đón nhận. Và như kế hoạch lúc đầu, mỗi năm STAR sẽ giới thiệu:

• Quyển 1: ra mắt vào cuối học kỳ 1, hướng tới kỳ thi HSG quốc gia.

• Quyển 2: ra mắt vào cuối học kỳ 2, hướng tới kỳ thi tuyển sinh THPT, Đại học.

Trong tập san lần này, đối tượng chủ yếu được hướng đến là các em học sinh đang
chuẩn bị cho kỳ thi HSG quốc gia vào 3 ngày cuối tuần tới. Bên cạnh các chuyên đề,
tập san cũng có ba đề ôn tập dành cho các thí sinh.
Về nội dung, chúng tôi có đủ các bài viết cho 5 phân môn, đặc biệt ngoài lực lượng
giáo viên trẻ tại STAR EDUCATION, Ban biên tập còn nhận được bài viết của thầy
Kiều Đình Minh, GV Chuyên Hùng Vương, Phú Thọ và em Trương Tuấn Nghĩa, học sinh
THPT Chuyên KHTN Hà Nội. Rất mong nhận được thêm sự đóng góp của bạn đọc gần
xa cho tập san để ngày một phong phú hơn, phục vụ tốt hơn cho cộng đồng dạy và
học Toán.
Dự kiến số tiếp theo, Tập san 04, sẽ được xuất bản vào tháng 05 tới với nội dung chủ
yếu dành cho các bạn chuẩn bị thi tuyển sinh 10 và tuyển sinh ĐH, xin đón nhận các
bài viết gửi về cho Ban biên tập. Mọi đóng góp xin gửi về các địa chỉ nguyentangvu@
gmail.com hoặc lephuclu@gmail.com.
Bản quyền thuộc trung tâm STAR EDUCATION, được đăng tải miễn phí trên mạng.
Mong rằng tài liệu này sẽ được đón nhận và được chia sẻ rộng rãi. Xin chân thành
cảm ơn.

Tập san Toán học STAR EDUCATION


Công thức nội suy Lagrange

Vương Trung Dũng


(GV PTNK TP Hồ Chí Minh)

1. Giới thiệu
Cho đa thức P(x) có bậc nhỏ hơn n + 1 và n + 1 số thực phân biệt x i ; i = 1, n + 1. Khi
n+1 n+1
Q x−x j
đó P(x) được xác định duy nhất như sau: P(x) = P(x i ) ·
P
x i −x j (*)
i=1 j=1
j6=i

n+1 n+1
x−x j
Chứng minh. Xét đa thức Q(x) = P(x) − P(x i ) ·
P Q
x i −x j . Khi đó deg Q(x) ≤ n và
i=1 j=1
j6=i
Q(x i ) = 0 nên Q(x) có (n + 1) nghiệm x i . Do đó Q(x) ≡ 0.
Tính duy nhất của P(x)được suy ra ngay từ nhận xét rằng hai đa thức bậc nhỏ hơn
hoặc bằng n nhận giá trị bằng nhau tại (n + 1) điểm thì chúng trùng nhau.

Từ công thức trên, ta suy ra ngay kết quả quan trọng sau:
Cho đa thức P(x) có bậc không quá n và nhận giá trị hữu tỉ tại n + 1 số hữu tỉ khác
nhau. Chứng minh P(x) ∈ Q[x].
Công thức nội suy Lagrange có thể dùng để:

1. Tính giá trị của một đa thức tại một điểm (cho biết giá trị tại n + 1 điểm phân
biệt, tính giá trị tại điểm mới.

2. Tính tổng liên quan đến các đẳng thức tổ hợp. Chứng minh các BĐT.

3. Nội suy liên quan đến số hữu tỷ, đa thức nguyên.

2. Các bài toán chọn lọc

Bài toán 1. Cho n số phân biệt a1 , a2 , ..., an và đa thức P(x) có bậc deg P(x) ≤ n − 2.
Chứng minh rằng

P(a1 ) P(an )
T= + ... + = 0.
(a1 − a2 )(a1 − a3 )...(a1 − an ) (an − a1 )(an − a2 )...(an − an−1 )

3
4 VƯƠNG TRUNG DŨNG

Lời giải. Áp dụng công thức nội suy Lagrange ta được:

(x − a2 )(x − a3 )...(x − an ) (x − a1 )(x − a2 )...(x − an−1 )


P(x) = P(a1 ) + ... + P(an )
(a1 − a2 )(a1 − a3 )...(a1 − an ) (an − a1 )(an − a2 )...(an − an−1 )

Đồng nhất hệ số của x n−1 ở hai vế ta thu được T = 0 (đpcm).

Nhận xét. Từ đó, ta có cơ sở phương pháp hệ số bất định như sau:


Cho a1 , a2 , ..., an là n số thực đôi một phân biệt và deg f ≤ n − 1. Chứng minh rằng tồn
tại các số thực A1 , A2 , ..., An sao cho

f (x) A1 A2 An
= + + ... + .
(x − a1 )...(x − an ) x − a1 x − a2 x − an

Bài toán 2. India 2001 Cho a ≥ 3 và p(x) ∈ R[x], deg p = n. Chứng minh rằng

max{|a j − p( j)| : 0 ≤ j ≤ n + 1} ≥ 1.

Lời giải:
Đặt y j = p( j); j = 1, n + 1. Khi đó đa thức p(x) là đa thức duy nhất có bậc không quá
n + 1 và nhận giá trị bằng yk khi x = j nên theo công thức nội suy Lagrange
n+1
X (x − 0)...(x − ( j − 1))(x − ( j + 1))...(x − (n + 1))
p(x) = yj .
j=0
( j − 0)...( j − ( j − 1))( j − ( j + 1))...( j − (n + 1))

Vì d e g p = n nên hệ số của x n+1 là


n+1
X yj
= 0.
j=0
j!(n + 1 − j)!(−1)n+1− j

Nhân hai vế cho (n + 1)! ta được


n+1
X
C jn+1 (−1)n+1− j y j = 0. (∗)
j=0

Giả sử rằng a j − 1 < y j < a j + 1, ∀ j. Khi đó

(−1)n+1− j a j − 1 < (−1)n+1− j y j < (−1)n+1− j a j + 1, ∀ j.

n+1 n+1
C jn+1 (−1)n+1− j y j > C jn+1 ((−1)n+1− j a j − 1) = (a − 1)n+1 − 2n+1 ≥ (3 −
P P
Do đó
j=0 j=0
1)n+1 − 2n+1 = 0, mâu thuẫn với (*). Từ đó suy ra điều phải chứng minh.

m
= (m−k)n (−1)k Cn+1
P
Bài toán 3. Với mỗi cặp số nguyên m, n mà 1 ≤ m ≤ n, đặt Rm
n
k
.
k=0
Với các số nguyên như thế chứng minh rằng Rn−m+1
n
= Rm
n
.

Tập san Toán học STAR EDUCATION


VƯƠNG TRUNG DŨNG

Lời giải. Trước tiên để ý rằng 1 ≤ m ≤ n ⇒ 1 ≤ n − m + 1 ≤ n. Từ đó theo định nghĩa


Đặt
n−m+1 n−m+1
P € Šn
= (n − m + 1 − k)n (−1)k Cn+1 = (n + 1 − k) − m (−1)k Cn+1
P
Rn−m+1
n
k n+1−k

k=0 k=0

n+1
= (i − m)n (−1)n+1−i Cn+1 (thay i = n + 1 − k ⇒ k = n + 1 − i)
P i
i=m

n+1 n+1
= (m − i)n (−1)n (−1)n+1−i Cn+1 =− (m − k)n (−1)k Cn+1 (thay k = n + i).
P i
P k
i=m k=m
(1.1)
So sánh với công thức xác định của Rm
n
và chú ý rằng (m − k) n
(−1) k k
C n+1
khi k = m ta
n−m+1
thấy R n = R n khi và chỉ khi
m

n+1
X
(m − k)n (−1)k Cn+1
k
= 0. (2)
k=0

Để chứng minh (2) ta đặt P(x) = (m − x)n và các nút nội suy x j = j, 1 ≤ j ≤ n + 1, áp
dụng công thức nội suy Lagrange ta có
n+1
X
P(0) − P(k)ωk (0) = 0, (3)
k=1

(x − x i )
Q
i6=k
trong đó ωk (x) = Q . Nhưng ta cũng có
(x k − x i )
i6=k

Π(−i).k (n + 1)!
ωk (0) = = (−1)n Q = −(−1)k Cn+1
k
,
(k − i).k (k − i). (k − i)k
Q
i>k i>k

nên từ (3) ta suy ra được (2).

Bài toán 4. Cho p là số nguyên tố và P(x) ∈ Z[x] là một đa thức bậc d thoả mãn

1. P(0) = 0, P(1) = 1;

2. Với mọi số nguyên dương n thì số dư trong phép chia P(n) cho p là 0 hoặc 1.

Chứng minh rằng d ≥ p − 1.

Lời giải. Giả sử d ≤ p − 2. Áp dụng công thức nội suy Lagrange với p − 1 nút nội suy
x i ; i = 1, 2, ..., p − 2 ta được
p−2 p−2
X Y x−j
P(x) = P(i) .
i=0 j=0, j6=i
i − j

Tập san Toán học STAR EDUCATION


6 VƯƠNG TRUNG DŨNG

Suy ra
p−2
X
P(p − 1) = P(i).(−1) p−i C p−1
i
.
i=0
i
Do p nguyên tố nên C p−1 ≡ (−1)i (mod p), với mọi i = 1, 2, ..., p − 2. Do đó

p−2
X
P(p − 1) ≡ − P(i) (mod p).
i=0

Suy ra
p−1
X
P(i) ≡ 0(mod p).
i=0
p−1
P(i) ≡ k(mod p) với k = 1, 2, ..., p−1,
P
Nhưng từ hai điều điện của giả thiết ta lại có
i=0
vô lí. Vậy d ≥ p − 1.

Bài toán 5. Chứng minh rằng không tồn tại đa thức P ∈ R[x] có bậc dương sao cho
P(m) là số nguyên tố với mọi số nguyên dương m.

Lời giải. Giả sử tồn tại đa thức thoả yêu cầu đề bài khi đó ta chứng minh n!P(x) ∈
Z[x].
n n
x−j
Thật vậy, theo công thức nội suy Lagrange ta có P(x) =
P Q
P(i). nên
i=0 j=0, j6=i i − j

n
X P(i)C i (−1)n−i
n
n!P(x) = x(x − 1)...(x − n) ∈ Z[x].
i=0
x −i

Chọn hai số nguyên tố p, q sao cho p, q > n và P(a) = p, P(b) = q với a, b là hai số
nguyên dương nào đó. Khi đó theo định đí thặng dư Trung Hoa, tồn tại số nguyên
dương c sao cho
¨ ¨
c ≡ a(mod p) n!P(c) ≡ n!P(a) ≡ 0(mod p)

c ≡ b(mod q) n!P(c) ≡ n!P(b) ≡ 0(mod q).

Do đó n!P(c) ≡ 0(mod pq) Điều này là vô lí, vì P(c) là số nguyên tố và (n!, pq) = 1.

k
Bài toán 6. Cho đa thức P(x) có bậc n và thỏa mãn điều kiện P(k) = ;k =
k+1
0, 1, 2...n. Tính P(n + 1)

Lời giải. Áp dụng công thức nội suy Lagrange


n
X k x(x − 1)...(x − k + 1)(x − k − 1)...(x − n)
P(x) = .
k=0
k+1 k(k − 1)...1(−1)...(k − n)

Tập san Toán học STAR EDUCATION


VƯƠNG TRUNG DŨNG

Từ đó
n
k (n + 1)...(n − k + 2)(n − k)...1
P(x) =
P
k=0 k + 1 k(k − 1)...1(−1)...(k − n)(n − k + 1)

n
(n + 1)! 1 P n
= (−1)n−k k = (−1)n−k kCn+2
P k+1

k=1 (k + 1)!(n − k + 1)! n + 2 k=1

Bài toán 7. Cho đa thức P(x) có bậc không quá 2n và |P(x)| ≤ 1, ∀k ∈ {−n, −n +
1, ..., n − 1, n}. Chứng minh rằng

|P(x)| ≤ 4n , ∀x ∈ [−n, n].

Lời giải. Ta chứng minh


n
Y n
Y
|x − i| ≤ (2n)! và |x − i| = (n + k)!(n − k)!.
i=−n,i6=k i=−n,i6=k

Suy ra
n n
X (2n)! X
j
|P(x)| ≤ = C2n = 22n = 4n .
k=−n
(n + k)!(n − k)! j=0

1
Bài toán 8. (Rumani 1981) Cho đa thức P(x) có bậc n và thỏa mãn P(x) = k
,n =
Cn+1
0, 1, 2, ..., n. Tính P(n + 1).

Lời giải. Áp dụng công thức nội suy Lagrange cho P(x) tại n+1 nút nội suy là 0, 1, ..., n
ta được
n n
X € Y x − iŠ
P(x) = P(k)
k=0 i6=k,i=0
k−i
n
(k − i) = k(k − 1)...(k − k + 1)(k − k − 1)...(k − n) = k!(n − k)!(−1)n−k . Do đó
Q

k=0

n
X (−1)n−k (n − k + 1)! Y
P(x) = (x − i)],
k=0
(n + 1)!(n − k)!

suy ra X X
P(n + 1) = (−1) n−k
= (−1) j
(n + 1)!
(n + 1 − i) =
Q
(vì ). Dẫn đến
n+1−k
¨
0, n = 2m + 1
P(n + 1) =
1, n = 2m.

Tập san Toán học STAR EDUCATION


8 VƯƠNG TRUNG DŨNG

Nói cách khác:


n+1
¨
X 1, n = 2k
0= (−1)i Cn+1
i
P(i) = (−1)n+1 P(n + 1) +
i=0
0, n = 2k + 1.
¨
1, n = 2k
Từ đó P(n + 1) =
0, n = 2k + 1.

Bài toán 9. Cho đa thức P(x) = x n + a1 x n−1 + ... + an ∈ R[x]. Giả sử x 0 , x 1 , ..., x n là
các số nguyên thỏa x 0 > x 1 > ... > x n . Chứng minh rằng tồn tại k ∈ {0, 1, ..., n} sao
n!
cho |P(x k )| ≥ n .
2

Lời giải. Theo công thức nội suy Lagrange, ta có


n n
X Y x − xj
P(x) = P(x i ) ,
i=0 j6=i, j=0
xi − x j

so sánh hệ số bậc cao nhất ở hai vế ta được


n n
X Y P(x i )
1= .
i=0 j6=i, j=0
xi − x j

Vì x 0 , x 1 , ..., x n là một dãy giảm ngặt các số nguyên nên ta có


n i−1 n
|x j − x i | =
Q Q Q
|x j − x i | |x i − x j |
j6=i, j=0 j=0 j=i+1

1 i
≥ i!(n − i)! = C .
n! n
¦ ©
Đặt |P(x k )| = max |P(x 0 )|, |P(x 1 )|, ..., |P(x n )| . Theo bất đẳng thức tam giác ta có

n n
X |P(x i )| |P(x k )| X i 2n |P(x k )|
1≤ n ≤ C = .
Q n! i=0 n n!
i=0 |x j − x i |
j6=i, j=0

n!
Suy ra |P(x k )| ≥ .
2n

Bài toán 10. Giả sử a1 , a2 , ..., an là những số nguyên dương thoả mãn ai 6= a j khi
n aik
i 6= j. Chứng minh rằng với bất kì số nguyên dương k ta luôn có
P
cũng
(ai − a j )
Q
i=1
j6=i
là một số nguyên.

Tập san Toán học STAR EDUCATION


VƯƠNG TRUNG DŨNG

n
Lời giải. Xét hai đa thức f (x) = x k và g(x) = (x − ai ). Ta thực hiện phép chia
Q
i=1
f (x) = q(x)g(x) + r(x), trong đó deg r(x) ≤ n − 1, suy ra f (ai ) = r(ai ) với mọi
i = 1, 2, ..., n. Theo công thức nội suy Lagrange, ta có
n
X Y x − aj
r(x) = r(ai ) .
i=1 j6=i
ai − a j

Dễ thấy r(x) là đa thức hệ số nguyên và r(ai ) = f (ai ) = aik với mọi i = 1, 2, ..., n.
n ak
Q i
P
Hệ số cao nhất của r(x) là hệ số của x n−1 và bằng . Từ đây suy ra
i=1 (ai − a j )
i6= j
n
P aik
là một số nguyên.
(ai − a j )
Q
i=1
i6= j

Bài toán 11. Trong mặt phẳng toạ độ, một điểm được gọi là hỗn tạp nếu một trong
hai toạ độ của nó là số hữu tỉ còn toạ độ còn là là số vô tỉ. Tìm tất cả các đa thức mà
đồ thị của nó không chứa bất cứ một điểm hỗn tạp nào.

Lời giải. Ta gọi một đa thức là thuần khiết nếu nó không chứa bất cứ một điểm hỗn
tạp nào. Ta sẽ chứng minh chỉ có những chỉ có những đa thức bậc nhất hệ số hữu tỉ
là thoả yêu cầu đề bài.
Giả sử tồn tại P(x) = an x n + an−1 x n−1 +...+ a1 x + a0 , n ≥ 2 là một đa thức thuần khiết,
từ đó suy ra P(x) nhận giá trị hữu tỉ tại mọi giá trị hữu tỉ của biến x nên theo công
thức nội suy Lagrange P(x) ∈ Q[x].
Gọi m là mẫu chung lớn nhất của các ak thế thì P(mx) và −P(mx) là các đa thức
thuần khiết với hệ số nguyên và trong hai đa thức này có một đa thức có hệ số bậc
cao nhất là số nguyên dương. Do đó không mất tổng quát có thể giả sử an > 0 và ak
là các số nguyên.
Gọi p là số nguyên tố sao cho p không là ước của an . Gọi r là số nguyên đủ lớn sao
pr + 1
cho tồn tại số dương x 0 thoả mãn P(x 0 ) = (Vì hệ số bậc cao nhất của P là số
p
dương nên số r như thế là tồn tại). Vì P(x 0 ) là số hữu tỉ nên x 0 cũng là số hữu tỉ, ta
s
viết x 0 = , với (s, t) = 1. Khi đó
t
pr + 1 an s n + an−1 s n−1 t + ... + a0 t n
= P(x 0 ) =
p tn
hay
(pr + 1)t n = p(an s n + an−1 s n−1 t + ... + a0 t n ).
Vì p chia hết vế phải nên p cũng chia hết vế trái, dẫn đến p|t. Hơn nữa, vì n ≥ 2 vế
trái chia hết cho p2 , dẫn đến

p|(an s n + an−1 s n−1 t + ... + a0 t n ).

Vì p|t nên p chia hết an−1 s n−1 t + ... + a0 t n suy ra p|an s n .


Vì p|t và (s, t) = 1 nên p không là ước của s và do đó theo sự tồn tại của p ta có p

Tập san Toán học STAR EDUCATION


10 VƯƠNG TRUNG DŨNG

không là ước của an . Điều này cho ta p không là ước của an s n , mâu thuẫn. Từ đó suy
ra không có một đa thức thuần khiết nào với bậc lớn hơn 1.
Như vậy mọi đa thức thuần khiết có bậc là 0 hoặc 1. Mọi đa thức hằng đều có điểm
hỗn tạp. Dễ dàng kiểm tra mọi đa thức bậc nhất với hê số hữu tỉ đều thoả yêu cầu đề
bài. Ta có điều phải chứng minh.

Bài toán 12. (USAMO 2002) Cho F (x) ∈ R[x] là một đa thức monic có bậc n. Chứng
minh rằng F (x) là trung bình cộng của hai đa thức bậc n có đủ n nghiệm thực.

Lời giải. Gọi y1 , y2 , ..., yn là các số thực thoả


¨
yi < min{0, 2F (i)}, nếu i lẻ
yi > max{0, 2F (i)}, nếu i chẵn.

Theo công thức nội suy Lagrange tồn tại một đa thức bậc nhỏ hơn n sao cho P(i) = yi ,
với i = 1, 2, ..., n. Đặt

G(x) = P(x) + (x − 1)(x − 2)...(x − n) và H(x) = 2F (x) − G(x).

Khi đó G(x), H(x) là hai đa thức monic, hệ số thực có bậc n và có trung bình cộng là
F (x). Vì y1 , y3 , y5 , ... < 0 và y2 , y4 , y6 ... > 0, G(i) = yi , G(i + 1) = yi+1 trái dấu nhau
nên G(x) có nghiệm thuộc đoạn [i, i + 1], với mọi i = 0, 1, ..., n − 1. Do đó G(x) có
ít nhất n − 1 nghiệm, nghiệm còn lại cũng là nghiệm thực vì nếu ngược lại nó sẽ có
thêm một nghiệm phức liên hợp.
Tương tự, nếu i là số lẻ thì G(i) = yi < 2F (i) dẫn đến H(i) = 2F (i) − G(i) > 0, còn
nếu i là số chẵn thì G(i) = yi > 2F (i) dẫn đến H(i) = 2F (i)− G(i) < 0. Điều này cũng
cho ta H(x) có đủ n nghiệm thực như trong trường hợp của G(x).

Bài toán 13. Cho a1 , a2 , a3 , a4 , b1 , b2 , b3 , b4 là các số thực thoả bi − a j 6= 0, với mọi


i, j = 1, 2, 3, 4. Giả sử hệ phương trình

X1 X2 X3 X4

 + + + =1
b − a b − a b − a b − a

1 1 1 2 1 3 1 4






X1 X2 X3 X4


+ + + =1


 b2 − a1 b2 − a2 b2 − a3 b2 − a4

 X1 X2 X3 X4
+ + + =1



b3 − a1 b3 − a2 b3 − a3 b3 − a4







X4 X2 X3 X4


+ + + =1


b4 − a1 b4 − a2 b4 − a3 b4 − a4

có nghiệm duy nhất. Hãy tính X 1 + X 2 + X 3 + X 4 theo ai , bi .

Tập san Toán học STAR EDUCATION


VƯƠNG TRUNG DŨNG

4 4
Lời giải. Đặt P(x) = (x − ai ) − (x − bi ). Khi đó hệ số của x 3 trong P(x) là
Q Q
i=1 i=1
4
X 4
X
bi − ai .
i=1 i=1

Theo công thức nội suy Lagrange


4 4
X Y x − aj
P(x) = P(ai ) .
i=1 j=1, j6=i
ai − a j
4
(x − a j ) là 1 nên hệ số của x 3 trong P(x) là
Q
Vì hệ số của x 3 trong tích
j=1, j6=i

4
X P(ai )
4
.
(a j − ai )
Q
i=1
j=1, j6=i

P(b j )
Để ý rằng n = b j −ai which are the denominator of the 4 given equations!
(b j − ai )
Q
j=1, j6=i
Với j = 1, 2, 3, 4 cho x = b j vào trong công thức nội suy của P(x) và chia cả hai vế
cho P(b j ) ta được
4 4 4
X P(ai ) Y b j − ai X P(ai )
1= = .
P(b j ) j=1, j6=i a j − ai 4
(b j − ai ) (ai − a j )
Q
i=1 i=1
j=1, j6=i

P(ai )
So sánh với hệ phương trình đã cho, vì tính duy nhất nghiệm nên ta có X i = n ,
(ai − a j )
Q
j6=i, j=1
với i = 1, 2, 3, 4. Do đó
4 4 4 4
X X P(ai ) X X
Xi = 4
= bi − ai .
(ai − a j )
Q
i=1 i=1 i=1 i=1
j=1, j6=i

Bài toán 14. Cho các số thực a1 , a2 , ..., an và đa thức P(x) = (a−a1 )(x −a2 )...(x −an ).
Chứng minh rằng tồn tại số nguyên dương c sao cho

max{|P(x)| : 0 ≤ x ≤ 2} ≤ c n .max{|P(x)| : 0 ≤ x ≤ 1}.

i
Lời giải. Đặt S = max{|P(x)| : 0 ≤ x ≤ 1}. Với mỗi i = 0, 1, ..., n đặt bi = và
n
f i (x) = (x − b0 )...(x − bi−1 )(x − bi+1 )...(x − bn ). Theo công thức nội suy Lagrange ta
có n
X f i (x)
P(x) = P(bi ) .
i=0
f i (bi )

Tập san Toán học STAR EDUCATION


12 VƯƠNG TRUNG DŨNG

Với mỗi w ∈ [0, 2], |w − bk | ≤ |2 − bk |, với k = 0, 1, ..., n nên


n
i
| f i (w)| ≤ | f i (2)| = (2 − )
Q
i=0 n

2n(2n − 1)(2n − 2)...(n + 1) (2n)!


= = .
nn n!.nn
Do đó |P(bi )| ≤ S và
i!(n − i)!
| f i (bi )| = .
nn
Theo bất đẳng thức tam giác
n n
X f i (w) X
|P(w)| ≤ |P(bi )|. ≤S i
C2n n
.C2n−i .
i=0
f i (bi ) i=0

Từ đó ta có n n
X X
i
C2n n
C2n−i ≤ i
C2n n
C2n = 22n C2n
n
≤ 24n .
i=0 i=0
Suy ra
max{|P(w)| : 0 ≤ w ≤ 2} ≤ 24n S = 16n max{|P(x)| : 0 ≤ x ≤ 1}.

Bài toán 15. Tìm tất cả các đa thức P(x) ∈ R[x] sao cho với mọi số hữu tỉ r phương
trình P(x) = r có nghiệm hữu tỉ.

Lời giải. Dễ thấy những đa thức bậc nhất P(x) = ax + b ∈ Q[x] thỏa yêu cầu đề bài.
Ta sẽ chỉ ra rằng chỉ có những đa thức như vậy. Thật vậy, gọi P(x) là đa thức thỏa yêu
cầu đề bài, deg P(x) = n. Với mỗi r = 0, 1, 2, ..., n gọi x r ∈ Q sao cho P(x r ) = r. Theo
công thức nội suy Lagrange
n n
X Y x − xi
P(x) = r .
x − xi
r=0 i6= r,i=0 r

Khai triển vế phải dễ thấy P(x) có hệ số hữu tỉ. Gọi M là mẫu thức chung của các hệ
số và đặt Q(x) = M P(x) là đa thức có hệ số nguyên. Gọi k là hệ số bậc cao nhất của
Q(x) và c là hệ số tự do của P(x). Gọi p1 , p2 , p3 ... là một dãy các số nguyên tố. Khi đó
pi
phương trình P(x) = c + có nghiệm t i ∈ Q. Khi đó đa thức Q(x) − (cM + pi ) có hệ
M
số bậc cao nhất là k còn hệ số tự do là −pi .
1 pi
Vì Q(t i ) = 0 nên t i = hoặc t i = (với di là một ước của k). Vì các giá trị P(t i ) là
di di
1
phân biệt nên các giá trị của t i cũng phân biệt. Suy ra t i nhận giá trị nhiều nhất là
di
pi
bằng số ước của k, điều này dẫn đến tồn tại d|k sao cho có vô hạn lần t i = . Điều
d
dx
này cho ta P(x) − (c + ) có vô hạn nghiệm.
M
d
Vậy P(x) = ax + b, với a = 6= 0 và b, c ∈ Q.
M

Tập san Toán học STAR EDUCATION


VƯƠNG TRUNG DŨNG

3. Bài tập tự luyện

Bài 1. Với mọi 1 ≤ j ≤ 2019 đặt


2019
Y
bj = j 2019
(i 2019 − j 2019 ).
i=1,,i6= j

Tính
1 1 1
+ + ... + .
b1 b2 b2019

Bài 2. Cho P(x) là đa thức monic bậc n > 1. Biết rằng P(x) có n nghiệm thực
x 1 , x 2 , ..., x n phân biệt và khác 0. Chứng minh rằng
n
X 1 (−1)n+1
= .
i=1
x i P 0 (x i ) x 1 x 2 ...x n

Bài 3. Bết f (x) và g(x) là hai đa thức bậc 2014 sao cho f (n) + (−1)n g(n) = 2n , với
mọi n = 1, 2, ..., 4030. Tìm hệ số của x 2014 trong đa thức g(x).

Bài 4. Cho P(x) là đa thức bậc không quá 2017 và thỏa mãn |P(k)| ≤ k + 1, với mọi
k ∈ {0, 1, ..., 2017}. Chứng minh rằng |P(2018)| + |P(−1)| ≤ 2019(22018 − 1).

Bài 5. Chứng minh rằng không tồn tại đa thức P(x) ∈ R[x] với bậc n ≥ 1 sao cho
P(m) là số nguyên tố với mọi số nguyên dương m.

Bài 6. Chứng minh rằng với mỗi số nguyên dương n tồn tại đa thức P(x) ∈ Z[x] bậc
n sao cho P(0), P(1), ..., P(n) phân biệt và các số đó đều có dạng 2.2019k + 3, k ∈ N.

Bài 7. Cho đa thức P(x) = a0 + a1 x + a2 x 2 + ... + an x n . Đa thức P được gọi là đa


thức nhỏ nếu thỏa mãn đồng thời

(i) Với mọi i = 1, 2, .., n ta đều có 0 ≤ ai < 1.

(ii) P(x) là số nguyên với mọi x.

Có bao nhiêu đa thức nhỏ có bậc không quá n?

Tập san Toán học STAR EDUCATION


14 VƯƠNG TRUNG DŨNG

Bài 8. Cho p là một số nguyên tố. Tìm số nguyên dương n nhỏ nhất sao cho tồn
tại đa thức P(x) bậc n thỏa mãn P(0) = 0, P(1) = 1 và với mọi λ ∈ N thì p |
(P(λ) − 2)(P(λ) − 1)P(λ).

Bài 9. Cho p > 2 là một số nguyên tố và f là đa thức hệ số nguyên có bậc p − 1. Giả


sử với mọi số nguyên a, b nếu a − b không chia hết cho p thì f (a) − f (b) cũng không
chia hết cho p. Chứng minh rằng hệ số cao nhất của f chia hết cho p.

Bài 10. Cho dãy (εn )n∈N , trong đó εi = ±1. Chứng minh rằng dãy số này tuần hoàn
với chu kì là lũy thừa của 2 khi và chỉ khi εn = (−1) P(n) , trong đó P(x) ∈ Q[x] là một
đa thức giá trị nguyên (đa thức giá trịnguyên là đa thức thỏa P(n) ∈ Z, ∀n ∈ Z).

Bài 11. Cho hai số nguyên t, n không nhỏ hơn 2. Tìm số nguyên m lớn nhất sao cho
tồn tại đa thức P(x) ∈ Q[x] có bậc n thỏa mãn : Với mỗi số nguyên k ∈ {1, 2, ..., m}
P(k) P(k)
thì k là số nguyên còn k+1 không là số nguyên.
t t

Bài 12. Cho đa thức f (x) ∈ R[x] mà không là hằng số. Chứng minh rằng với
mỗi số c > 0, tồn tại số nguyên dương n0 thỏa mãn điều kiện: Nếu đa thức monic
P(x) ∈ R[x] có bậc k ≤ n0 thì các số nguyên x thỏa mãn | f (P(x))| ≤ c không vượt
quá k.

Tập san Toán học STAR EDUCATION


Tài liệu tham khảo

[1] Nguyễn Duy Thái Sơn, 2017 Một số bài toán liên quan đến chủ đề đa thức - Bài
giảng tập huấn giáo viên chuyên toán.

[2] Phạm Viết Huy, Tính chất số học trong một số bài toán đa thức.

[3] Kim Y.Li, Lagrange Interpolation Formula.

[4] Huỳnh Kim Linh, Nguyễn Minh Tuấn, Doãn Quang Tiến, 2019, Chuyên đề đa
thức và số học - Page "Tạp chí Tư liệu và toán học".

15
Một số kĩ thuật giải phương trình hàm trên
tập R+

Trần Bá Đạt
(GV THPT Gia Định, TP Hồ Chí Minh)

1. Bài toán mở đầu


Phần này trình bày hai bài toán hàm số, một bài yêu cầu tìm hàm xác định trên R
và một bài yêu cầu tìm hàm xác định trên R+ . Qua đó, ta sẽ nhận thấy được những
thuận lợi và khó khăn của việc giải phương trình hàm trên R+ .

Bài toán 1. Tìm tất cả các hàm f : R −→ R thỏa mãn

(i) f (x + f ( y)) = 2 y + f (x), với mọi x, y ∈ R.

(ii) Với mọi x > 0, tồn tại y < 0 thỏa mãn f ( y) = x.

f (0)
Lời giải. Đặt a = − . Trong điều kiện thứ nhất, cho x = 0 và y = a, ta có
2
f ( f (a)) = 0.
Thay x = 0, y = f (a) vào đề bài, ta lại có
2 f (a) = 0.
Do đó f (a) = 0, kéo theo f (0) = f ( f (a)) = 0. Thay x = 0 vào phương trình đề cho,
ta lại có
f ( f ( y)) = 2 y, với mọi y ∈ R.
Lấy f hai vế, ta suy ra
2 f ( y) = f (2 y), với mọi y ∈ R.
y
Thay y bởi f vào phương trình đề cho, ta suy ra
2
   y 
f (x + y) = f x + f f
y2 
= f (x) + 2 f
2
= f (x) + f ( y), với mọi x, y ∈ R.

16
TRẦN BÁ ĐẠT

Mặt khác, từ điều kiện thứ hai, với mọi x > 0, tồn tại y < 0 thỏa f ( y) = x nên

f (x) = f ( f ( y)) = 2 y < 0.

Nói cách khác, f < 0 trên (0, +∞). Kết hợp với tính cộng
p tính, ta suy ra f đơn điệu
giảm, kéo theo f (x) = ax. Thử lại, ta nhận f (x) = − 2x làm nghiệm duy nhất của
bài toán.

Bài toán 2 (Gặp gỡ Toán học 2019). Tìm tất cả các hàm f : R+ −→ R+ thỏa mãn

f (x + f ( y)) = 2 y + f (x), với mọi x, y ∈ R+ .

Lời giải.
Nhận xét. f đơn ánh.
Lấy a > b > 0 thỏa f (a) = f (b). Lần lượt thay y = a và y = b vào phương trình, ta

f (x + f (a)) = 2a + f (x),

f (x + f (b)) = 2b + f (x), với mọi x > 0.
Do đó a = b, tức f đơn ánh.
Nhận xét. f (2x) = 2 f (x) với mọi x > 0.
Thay x bởi x + f ( y) vào phương trình đề cho, ta có

f (x + 2 f ( y)) = 2 y + f (x + f ( y))
= 2 y + 2 y + f (x)
= 4 y + f (x), với mọi x, y > 0.

Mặt khác, thay y bởi 2 y vào phương trình đề cho, ta cũng có

f (x + f (2 y)) = 4 y + f (x), với mọi x, y > 0.

Do đó

f (x + 2 f ( y)) = f (x + f (2 y)), với mọi x, y > 0.

Áp dụng tính đơn ánh của f , ta thu được

2 f ( y) = f (2 y), với mọi y > 0.

Nhận xét. f ( f ( y)) = 2 y với mọi y > 0.


Thay x = f ( y) và y bởi 2 y vào phương trình đề cho, ta có

f ( f ( y) + f (2 y)) = 4 y + f ( f ( y)) ,

hay
f (3 f ( y)) = 4 y + f ( f ( y)) , với mọi y > 0. (1)

Tập san Toán học STAR EDUCATION


18 TRẦN BÁ ĐẠT

Mặt khác, thay x = 2 f ( y) vào phương trình đề cho, ta cũng có

f (3 f ( y)) = 2 y + 2 f ( f ( y)), với mọi y > 0. (2)

Từ (1) và (2), ta thu được

f ( f ( y)) = 2 y, với mọi y > 0.


y
Thay y bởi f vào phương trình đề cho, ta được
2
   y  y
f x+f f = 2f + f (x),
2 2
hay
f (x + y) = f (x) + f ( y), với mọi x, y > 0.
Mà f > 0 trên (0, +∞) nên f tăng ngặt.
p Do đó f (x) = ax với mọi x > 0. Thay vào
phương trình đề cho, ta tìm được a = 2. p
Vậy phương trình có nghiệm duy nhất f (x) = 2x.

Cả hai bài toán trên đều được khai thác từ đẳng thức f (x + f ( y)) = 2 y + f (x). Tuy
nhiên, bài toán đầu tiên yêu cầu tìm hàm xác định trên R còn bài toán thứ hai yêu
cầu tìm hàm trên R+ . Việc xử lí bài toán thứ hai (trên R+ ) khó khăn hơn bài toán thứ
nhất (trên R) vì các phép thế yêu cầu chặt chẽ hơn. Đặc biệt, với phương trình hàm
trên R+ , ta không được phép thế giá trị 0, một phép thế xuất hiện ở hầu hết các bài
phương trình hàm trên R.
Ngược lại, việc tìm hàm số xác định và nhận giá trị trên R+ có một điểm lợi là nếu ta
chứng minh được hàm số đã cho cộng tính thì ta sẽ thu được tính tuyến tính của hàm
số, thay vì phải chứng minh thêm các yếu tố phụ (đơn điệu, liên tục, bị chặn) như các
bài toán hàm số trên R.
Vì lí do đó, bài viết này trình bày một số bài toán hàm số trên R+ , đồng thời đưa ra
lời giải và một số kĩ thuật thường dùng đối với những bài toán như vậy.

2. Kiến thức chuẩn bị


Đầu tiên, ta trình bày một tính chất hàm thường được khai thác trong các bài toán
hàm số trên R+ và liên hệ giữa nó với hàm tuần hoàn.

Định nghĩa 1 (Hàm trên tuyến tính). Hàm số f : R+ −→ R+ được gọi là trên tuyến
tính nếu tồn tại a > 0 thỏa

f (x) ≥ ax, với mọi x > 0.

Định nghĩa 2. Cho X , Y ⊂ R và f : X −→ Y và số thực dương T. Hàm f được gọi là


tuần hoàn với chu kì T (gọi tắt là tuần hoàn) nếu với mọi x ∈ X thì x + T ∈ X và

f (x + T ) = f (x).

Tập san Toán học STAR EDUCATION


TRẦN BÁ ĐẠT

Định nghĩa 3. Cho hàm số f : R+ −→ R+ là hàm trên tuyến tính. Khi đó f không
tuần hoàn.

Lời giải. Vì f trên tuyến tính nên tồn tại hệ số a > 0 thỏa

f (x) ≥ ax, với mọi x > 0.

Giả sử f tuần hoàn với chu kì T > 0. Lấy x > 0 bất kì, ta có

f (x) = f (x + nT ) ≥ ax + anT, với mọi n ∈ N.

Cho n tiến tới +∞, ta có điều vô lí. Do đó T = 0.


Bên cạnh đó, nếu hàm f : R+ −→ R+ là cộng tính, ta cũng suy ra f (x) = ax nhờ kết
quả f cộng tính và f > 0 trên (0, +∞) thì f (x) = ax.

Tính chất 1. Cho hàm f : R+ −→ R+ là cộng tính. Khi đó f (x) = ax với mọi x > 0.

3. Một số kĩ thuật thường dùng

3.1. Chứng minh tính trên tuyến tính


Đầu tiên, ta đến với một bài toán xuất hiện tính trên tuyến tính và cách áp dụng đặc
trưng này để giải phương trình hàm.

Bài toán 3 (Iran 2018). Cho hàm số f : R+ −→ R+ thỏa mãn

x + f ( y) 1 1
  ‹‹
=f +f , với mọi x, y > 0.
x f ( y) y x

a) Chứng minh rằng f (x) ≥ x với mọi x > 0.

b) Tìm f .

Lời giải. a) Phương trình được viết lại thành


1 1 1 1
  ‹‹
+ =f +f , với mọi x, y > 0.
f ( y) x y x
1
Thay x bởi , ta được
x
1 1
 ‹
+x= f + f (x) , với mọi x, y > 0.
f ( y) y
1
Giả sử tồn tại α > 0 sao cho f (α) < α. Thay x = α và y = , ta được
α − f (α)
1
+ α = f (α),
f (α − f (α))

Tập san Toán học STAR EDUCATION


20 TRẦN BÁ ĐẠT

hay

f (α) > α.

Điều này mâu chuẫn với f (α) < α. Do đó f (x) ≥ x với mọi x > 0.

b) Khi đó ta có
1 1 1 1
 ‹
x+ =f + f (x) ≥ + f (x) ≥ + x, với mọi x, y > 0.
f ( y) y y y

Do đó
1 1
≥ , với mọi y > 0,
f ( y) y

hay f (x) ≤ x với mọi x > 0.Kết hợp với f (x) ≥ x với mọi x > 0, ta suy ra
f (x) = x với mọi x > 0. Thử lại, ta nhận f (x) = x là nghiệm duy nhất của
phương trình.

Trong bài toán này, ta đã chứng minh rằng f (x) ≥ x với mọi x > 0 và sử dụng nó để
chứng minh f (x) ≤ x với mọi x > 0, từ đó suy ra f (x) = x với mọi x > 0. Ta có thể
áp dụng kĩ thuật này để giải bài toán sau.

Bài toán 4 (Mohammad Jafari). Tìm hàm số f : R+ −→ R+ thỏa mãn

f ( f (x) + 2 y) = f (2x + y) + 2 y, với mọi x, y > 0.

Lời giải. Đầu tiên, ta chứng minh f (x) ≥ 2x với mọi x > 0 bằng phản chứng. Thật
vậy, nếu tồn tại α > 0 thỏa f (α) < 2α thì thay x = α, y = 2α − f (α) , ta được

f (4α − f (α)) = f (4α − f (α)) + 2(2α − f (α)).

Điều này chứng tỏ f (α) = 2α (vô lí).


Như vậy f (x) ≥ 2x với mọi x > 0. Áp dụng vào đề bài, ta có

f (2x + y) + 2 y = f ( f (x) + 2 y)
≥ 2 f (x) + 4 y,

hay

f (2x + y) ≥ 2 f (x) + 2 y, với mọi x, y > 0.

Thay y bởi f ( y) trong bất đẳng thức trên, ta có

f (2x + f ( y)) ≥ 2 f (x) + 2 f ( y) ≥ f (x) + 2 f ( y) + 2x, với mọi x, y > 0.

Kết hợp với đẳng thức đề cho, ta lại có

f (2 y + x) + 2x = f ( f ( y) + 2x) ≥ f (x) + 2 f ( y) + 2x,

Tập san Toán học STAR EDUCATION


TRẦN BÁ ĐẠT

hay tương đương

f (2 y + x) ≥ f (x) + 2 f ( y), với mọi x, y > 0. (2.1)

Thay x bởi f (x) trong bất đẳng thức trên và áp dụng phương trình đề cho, ta lại có

f (2x + y) + 2 y = f ( f (x) + 2 y)
≥ f ( f (x)) + 2 f ( y)
≥ f ( f (x)) + f ( y) + 2 y,

hay

f (2x + y) = f ( f (x)) + f ( y), với mọi x, y > 0.

Do đó f là hàm tăng. Theo (2.1), ta có

f (2x + y) + 2 y = f ( f (x) + 2 y)
≥ 2 f ( y) + f ( f (x))
≥ f ( y) + 2 y + f (2x),

hay

f (2x + y) ≥ f (2x) + f ( y), với mọi x, y > 0.

Từ đây ta suy ra f trên cộng tính. Áp dụng vào phương trình đề bài, ta lại có

f (2x + y) + 2 y = f ( f (x) + 2 y)
≥ f ( f (x) + y) + f ( y)
≥ f ( f (x) + y) + 2 y,

hay

f (2x + y) ≥ f ( f (x) + y), tương đương f (x) ≤ 2x, với mọi x > 0.

Vậy f (x) = 2x với mọi x > 0.

Bài toán sau đây không sử dụng tính trên tuyến tính của hàm số nhưng vẫn mang tư
1 1
tưởng tương tự, tức chứng minh lần lượt f (x) ≥ và f (x) ≤ .
x x

Bài toán 5 (Tây Ban Nha 2018). Tìm tất cả các hàm số f : R+ −→ R+ thỏa mãn

f (x + f ( y)) = y f (x y + 1), với mọi x, y > 0.

Lời giải. a) Giả sử tồn tại α > 1 sao cho f (α) > 1 hoặc tồn tại α < 1 sao cho
1 − f (α)
f (α) < 1. Thay y = α, x = vào phương trình của đề bài, ta suy ra
1−α
α = 1 (Vô lí).
Vậy f (x) ≥ 1 với mọi x < 1 và f (x) ≤ 1 với mọi x > 1.

Tập san Toán học STAR EDUCATION


22 TRẦN BÁ ĐẠT

y −1
b) Với y > 1, thay x = vào phương trình đề cho, ta có
y
y −1
 ‹
f + f ( y) = y f ( y), với mọi y > 1.
y

Ta chứng minh rằng y f ( y) = 1 với mọi y > 0.

– Nếu tồn tại y > 1 thỏa y f ( y) > 1, ta suy ra


y −1
 ‹
f + f ( y) > 1,
y
kéo theo
y −1
+ f ( y) < 1.
y

Điều này tương đương với y f ( y) < 1 (vô lí).


– Nếu tồn tại y > 1 thỏa y f ( y) < 1, ta suy ra
y −1
 ‹
f + f ( y) < 1,
y
kéo theo
y −1
+ f ( y) > 1.
y

Điều này tương đương với y f ( y) > 1 (vô lí).


1
Do vậy f ( y) = với mọi y > 1. Với y ≤ 1, thay x = 1 vào phương trình đề
y
cho, ta có

f (1 + f ( y)) = y f ( y + 1).

Do f ( y) + 1 và y + 1 đều lớn hơn 0 nên ta suy ra


1 y
= .
1 + f ( y) y +1
1 1
Điều trên tương đương với f ( y) = . Vậy f ( y) = với mọi y > 0. Thử lại, ta
y y
nhận hàm này là nghiệm duy nhất của phương trình.

3.2. Thêm biến


Phương pháp thêm biến đặc biệt hữu hiệu trong các bài toán hàm số trên R+ . Tất
nhiên, để hoàn thiện một bài toán, ta thường dùng phương pháp thêm biến kết hợp
với một số phương pháp đi kèm như đối xứng, hoặc tính bằng hai cách. Đầu tiên, ta
trình bày lại lời giải cho bài toán mở đầu bằng phương pháp thêm biến và tính bằng
hai cách.

Tập san Toán học STAR EDUCATION


TRẦN BÁ ĐẠT

Bài toán 6 (Gặp gỡ Toán học 2019). Tìm tất cả các hàm f : R+ −→ R+ thỏa mãn

f (x + f ( y)) = 2 y + f (x), với mọi x, y ∈ R+ .

Lời giải. Ta dễ dàng chứng minh được f đơn ánh dựa vào định nghĩa. Trong phương
trình đề cho, thay y bởi y + z, ta có
f (x + f ( y + z)) = 2( y + z) + f (x)
= f (x + f ( y)) + 2z
= f (x + f ( y) + f (z)), với mọi x, y, z > 0.
Theo tính đơn ánh, ta suy ra
f (x + y) = f (x) + f ( y), với mọi x, y > 0.
p
Do đó f (x) = ax với mọi x > 0. Thử lại, ta nhận f (x) = 2x làm nghiệm duy nhất
của phương trình.
Bài toán sau đây cũng có thể xử lí thêm biến bằng cách thay y bởi y + f (z).

Bài toán 7 (IMO Shortlist 2007). Tìm hàm số f : R+ −→ R+ thỏa mãn

f (x + f ( y)) = f (x + y) + f ( y), với mọi x, y > 0.

Lời giải. Trong phương trình đề bài, thay y bởi y + f (z), vế trái trở thành
f (x + f ( y + f (z)) = f (x + f ( y + z) + f (z))
= f (x + f ( y + z) + z) + f (z)
= f (x + y + 2z) + f ( y + z) + f (z), với mọi x, y, z, > 0.
Vế phải của biểu thức ban đầu cũng trở thành
f (x + y + f (z)) + f ( y + f (z)) = f (x + y + z) + f (z) + f ( y + z) + f (z), với mọi x, y, z > 0.
Vì vế trái bằng vế phải, so sánh hai đẳng thức vừa có được, ta suy ra
f (x + y + 2z) = f (x + y + z) + f (z), với mọi x, y, z > 0
hay
f (x + y) = f (x) + f ( y), với mọi x > y > 0.
Với x, y > 0 bất kì. Lấy z > x + y, ta có
f (x + y + z) = f (x + y) + f (z), với mọi z > x, y > 0.
Mặt khác
f (x + y + z) = f (x) + f ( y + z) = f (x) + f ( y) + f (z), với mọi z > x, y > 0.
Do đó
f (x + y) = f (x) + f ( y), với mọi x, y > 0
Điều này chứng tỏ f cộng tính trên R+ nên f (x) = ax với mọi x > 0. Thay lại vào đề
bài, ta nhận f (x) = 2x làm nghiệm duy nhất của bài toán.

Tập san Toán học STAR EDUCATION


24 TRẦN BÁ ĐẠT

Ngoài cách xử lí trên, ta còn có một hướng xử lí khác cho bài toán này bằng việc quan
sát thấy sự độc lập giữa các biểu thức f ( y), y đối với x.

Lời giải. Hiển nhiên f (x) ≥ x với mọi x > 0. Thay x bởi x − y vào phương trình đề
bài, ta có

f (x + f ( y) − y)) = f (x) + f ( y), với mọi x > y > 0.

Thay y bởi y + f (z) − z vào đề bài, ta sẽ có

f (x + f ( y + f (z) − z)) = f (x + y + f (z) − z) + f ( y + f (z) − z), với mọi x > 0, y > z > 0.

Áp dụng đề bài vào cả hai vế, ta sẽ thu được

f (x + f ( y) + f (z)) = f (x + y) + f (z) + f ( y) + f (z)


= f (x + y) + f ( y) + 2 f (z), với mọi x > 0, y > z > 0.

Mặt khác, thay x bởi x + f (z) vào đề bài, ta cũng có

f (x + f (z) + f ( y)) = f (x + f (z) + y) + f ( y), với mọi x > 0, y > z > 0.

So sánh hai đẳng thức vừa có, ta suy ra

f (x + y + f (z)) = f (x + y) + 2 f (z), với mọi x > 0, y > z > 0.

Mà f (x + y)+2 f (z) = f (x + y + f (z)−z)+ f (z) = f (x + y +2( f (z)−z)), với mọi x >


0, y > z > 0 nên ta suy ra

f (x + y + 2( f (z) − z)) = f (x + y + f (z)), với mọi x > 0, y > z > 0.

Mà f trên tuyến tính nên không tuần hoàn, kéo theo

2( f (z) − z) = f (z),

hay f (z) = 2z với mọi z > 0. Thử lại, ta nhận đây là nghiệm hàm duy nhất của bài
toán.

Bài toán sau đây cũng giải được bằng cách thêm biến.

Bài toán 8 (Hà Nam 2019). Tìm tất cả các hàm số f : R+ −→ R+ thỏa mãn

1 1 f ( y)
  ‹‹
f ( y) f +f =1+ , với mọi x, y > 0.
y x x

1
Lời giải. Thay x bởi , bài toán được viết lại thành
x
1 1
 ‹
f + f (x) = x + , với mọi x, y > 0.
y f ( y)

Tập san Toán học STAR EDUCATION


TRẦN BÁ ĐẠT

1 1
 ‹
Từ đây ta suy ra f đơn ánh, f (x) ≥ x với mọi x > 0. Thay x bởi + f , ta có
x z
1 1 1 1 1 1
  ‹‹  ‹
f + +f = +f + ,
y x z x z f ( y)
hay
1 1 1 1 1 1
 ‹  ‹
f + + = +f + , với mọi x, y, z > 0.
y z f (x) x z f ( y)
Đổi chỗ y và z, ta suy ra
1 1 1 1
 ‹  ‹
f + =f + ,
z f ( y) y f (z)
tức
1 1
 ‹
f = + c, với mọi y > 0.
y f ( y)
Ta có
1 1
f (x) =  ‹ +c = + c,
1 1
f +c
x f (x)
hay
c([ f (x)]2 − c f (x) − 1) = 0, với mọi x > 0.

Vì f không bị chặn trên nên tồn tại x 0 sao cho [ f (x 0 )]2 − c f (x 0 ) − 1 > 0. Từ đó ta suy
ra c = 0. Do vậy
1 1
 ‹
f = , với mọi x, y > 0.
x f (x)
Khi đó bài toán trở thành
f ( y + f (x)) = x + f ( y), với mọi x, y > 0.

Tiếp tục thay y bởi y + f (z), ta suy ra


f ( y + f (z) + f (x)) = x + f ( y + f (z)) = x + z + f ( y) = f ( y + f (x + z)), với mọi x, y, z > 0.
Sử dụng tính đơn ánh, ta suy ra
f (x) + f (z) = f (x + z), với mọi x, z > 0.

Từ đây suy ra f cộng tính trên R+ . Kéo theo f (x) = ax với mọi x > 0. Thay vào đề
bài, ta suy ra f (x) = x là nghiệm duy nhất của phương trình.
Ta nhận thấy rằng bài toán này chính là Bài toán 3. Tuy nhiên, việc nhìn bài toán ấy
từ các góc độ khác nhau đã đưa ra cho ta lời giải mang thêm những kĩ thuật mà có thể
sử dụng được cho những bài toán khác. Bài toán dưới đây là một biến thể khác của
hai bài toán được đề cập ở trên. Tất nhiên, cách giải dưới đây cũng hoàn toàn khác và
yêu cầu sử dụng tính chất hàm cộng tính, bị chặn dưới trên một khoảng mở thì tuyến
tính.

Tập san Toán học STAR EDUCATION


26 TRẦN BÁ ĐẠT

Bài toán 9 (Áo 2018). Cho α là số thực khác 0. Tìm tất cả các hàm f : R+ −→ R+
thỏa mãn
1
f ( f (x) + y) = αx +  ‹ , với mọi x, y > 0.
1
f
y

Lời giải. Ta dễ dàng chứng minh được f đơn ánh. Thay y bởi f ( y), ta có
1
f ( f (x) + f ( y)) = αx + ‹ , với mọi x, y > 0. (2.2)
1

f
f ( y)

Thay đổi vai trò của x và y, ta suy ra


1 1
αx + ‹ = αy +  ‹,
1 1

f f
f ( y) f (x)
hay
1 1
‹ − αx =  ‹ − α y = c, với mọi x, y > 0.
1 1

f f
f (x) f ( y)
1
Thay ‹ = αx + c vào (2.2), ta có
1

f
f (x)

f ( f (x) + f ( y)) = αx + α y + 2c.

Khi đó với mọi x, y, z, t thỏa x + y = z + t thì ta có

f ( f (x) + f ( y)) = f ( f (z) + f (t)).

Theo tính đơn ánh ta suy ra f (x) + f ( y) = f (z) + f (t) với mọi x + y = z + t. Đặc biệt

f (x + 1) + f ( y + 1) = f (x + y + 1) + f (1), với mọi x > 0.

Đặt g(x) = f (x +1)− f (1) thì g bị chặn dưới, cộng tính trên (0, +∞) nên g(x) = ax,
suy ra f (x + 1) = ax + b. Do đó f (x) = c x + d với mọi x > 1. Thay vào đề bài, ta suy
ra d = 0, c = α = 1. Điều này chứng tỏ f (x) = x với mọi x > 1. Mặt khác, với x ≤ 1,
ta có

f (x) + f (3) = f (x + 1) + f (2),

suy ra

f (x) = x, với mọi x > 0.

Thử lại, ta nhận f (x) = x là nghiệm hàm duy nhất của phương trình.
Trong [2], tác giả cũng trình bày một số bài toán có cách giải tương tự.

Tập san Toán học STAR EDUCATION


TRẦN BÁ ĐẠT

Bài toán 10. Tìm tất cả các hàm f : R+ −→ R+ thỏa mãn

f (x + 3 f ( y)) = f (x) + f ( y) + 2 y, với mọi x, y > 0.

Lời giải. Thay y bởi y + 3 f (z), ta được


V T = f (x + 3 f ( y + 3 f (z))) = f (x + 3( f ( y) + f (z) + 2z))
= f (x + 6z + 3 f ( y) + 3 f (z))
= f (x + 6z + 3 f ( y)) + f (z) + 2z
= f (x + 6z) + f ( y) + 2 y + f (z) + 2z, với mọi x, y, z > 0.
Mặt khác,
V P = f (x) + f ( y + 3 f (z)) + 2 y + 6 f (z)
= f (x) + f ( y) + f (z) + 2z + 2 y + 6 f (z), với mọi x, y, z > 0.
Do đó
f (x + 6z) = f (x) + 6 f (z), với mọi x, z > 0.
Thay x bởi 6x rồi hoán đổi vai trò của x, z, ta suy ra
6 f (z) = f (6z) + c, với mọi z > 0.
Từ đó suy ra
f (x + 6z) = f (x) + f (6z) + c, với mọi x, z > 0.
Điều này chứng tỏ g(x) = f (x) + c là hàm cộng tính trên R+ nên tuyến tính. Thay lại
vào đề bài, ta suy ra f (x) = x với mọi x > 0.

Bài toán 11. Tìm tất cả các hàm f : R+ −→ R+ thỏa mãn

f (2x + 2 f ( y)) = x + f (x) + 2 y, với mọi x, y > 0.

Lời giải. Thay y bởi 2 y + 2 f (z), ta có


V T = f (2x + 2 f (2 y + 2 f (z))) = f (2x + 2( y + f ( y) + 2z))
= f (2x + 2 y + 4z + 2 f ( y))
= x + y + 2z + f (x + y + 2z) + 2 y, với mọi x, y, z > 0.
Mặt khác,
V P = x + f (x) + 2(2 y + 2 f (z)) = x + 4 y + f (x) + 4 f (z), với mọi x, y, z > 0.
Từ đó suy ra
f (x + y + 2z) + 2z = y + f (x) + 4 f (z), với mọi x, y, z > 0.
Đổi vài trò của x, y, ta suy ra
f (x) + y = f ( y) + x, với mọi x, y > 0.
Do đó f (x) = x + c, với mọi x > 0. Thay lại vào đề bài, ta suy ra f (x) = x là nghiệm
duy nhất của bài toán.

Tập san Toán học STAR EDUCATION


28 TRẦN BÁ ĐẠT

Tóm lại, khi đề bài có dạng f (x + g( y)) thì ta có thể thêm biến bằng cách thay y bởi
y + g(z) và biến đổi hai vế rồi so sánh.
Bên cạnh đó, hai bài toán sau trong [2] và [3] sử dụng tính đối xứng của các biến.
Đặc biệt ở bài toán thứ hai, ta cần dùng thêm tính chất hàm trên tuyến tính thì không
tuần hoàn để chứng minh đơn ánh.

Bài toán 12. Tìm tất cả các hàm f : R+ −→ R+ thỏa mãn

f (x f ( y)) − x = f (x y), với mọi x, > 0.

Lời giải. Thay x bởi f (x), ta có

f ( f (x) f ( y)) = f (x) + f ( f (x) y)


= f (x) + y + f (x y), với mọi x, > 0.

Hoán đổi vai trò của x, y ta suy ra

f (x) + y + f (x y) = f ( y) + x + f (x y), với mọi x, y > 0.

Do đó f (x) − x = f ( y) − y = c, hay f (x) = x + c với mọi x > 0. Thay vào đề bài, ta


suy ra f (x) = x + 1 là nghiệm duy nhất của bài toán.

Bài toán 13 (Mohammad Jafari). Tìm tất cả các hàm số f : R+ −→ R+ thỏa mãn

f (x + f (x) + y) = f (2x) + f ( y), với mọi x, y > 0.

Lời giải. Đầu tiên, ta chứng minh f đơn ánh. Lấy a, b thỏa mãn f (a) = f (b). Thay
lần lượt x = a, y = b và x = b, y = a vào phương trình đề cho, ta có

f (a + f (a) + b) = f (2a) + f (b), f (b + f (b) + a) = f (2b) + f (a).

Do đó f (2a) = f (2b). Lần lượt thay x = a, x = b vào phương trình đề cho, ta lại có

f (2a) + f ( y) = f (a + f (a) + y) = f (b + f (b) + y), với mọi y > 0.

Nếu a 6= b thì f tuần hoàn với chu kì T = |a − b|, khi x đủ lớn. Điều này mâu thuẫn
với f (x) ≥ x. Vậy f đơn ánh.
Thay y bởi 2 y vào phương trình đề cho và sử dụng tính đối xứng của hai vế, ta suy ra

f (x + f (x) + 2 y) = f ( y + f ( y) + 2x),

hay

f (x) − x = f ( y) − y = c, với mọi x, y > 0,

Thử lại f (x) = x +c vào đề bài, thấy thỏa. Vậy phương trình có nghiệm là f (x) = x +c,
với c > 0.
Cuối cùng là bài toán vừa được sử dụng làm đề thi Trường Đông Miền Nam 2019, yêu
cầu dùng tính đối xứng và toàn ánh trên một khoảng mở.

Tập san Toán học STAR EDUCATION


TRẦN BÁ ĐẠT

Bài toán 14 (Trường Đông Miền Nam 2019). Tìm tất cả các hàm f : R+ −→ R+
thỏa mãn

f (x f ( y) + f (x)) = 2 f (x) + x y, với mọi x, y > 0.

Lời giải. Đặt a = f (1). Trong phương trình đề cho, thay x = 1, ta được

f ( f ( y) + a) = 2a + y, với mọi y > 0.

Điều này chứng tỏ f toàn ánh trên (2a, +∞). Tiếp tục thay y bởi f ( y)+a vào phương
trình trên, ta suy ra

f ( f ( f ( y) + a) + a) = 2a + f ( y) + a,

hay

f ( y + 3a) = y + 3a, với mọi y > 0.

Bằng quy nạp, ta chứng minh được

f ( y + 3na) = y + 3na, với mọi y > 0, n ∈ N.

Trong phương trình đề cho, thay y bởi y + 3a và kết hợp với phương trình đó, ta có

f (x f ( y) + f (x) + 3ax) = 2 f (x) + x y + 3ax, với mọi x, y > 0.

Với mỗi x, z > 0, tồn tại n ∈ N đủ lớn và y > 0 thỏa x f ( y) + f (x) = z + 3na, ta suy ra

f (z + x) = f (z) + x = f (x) + z, với mọi x, z > 0.

Do đó f (x) = x + c. Thử lại ta suy ra f (x) = x + 1 là nghiệm duy nhất.

Ngoài ra, Bài toán 11 cũng có thể được xử lí tương tự như bài toán trên. Phần xử lí
này xin dành cho bạn đọc.

4. Một số bài toán áp dụng


Cuối cùng là một số bài tập áp dụng các kĩ thuật vừa nêu ra trong phần trước.

Bài toán 15. Tìm tất cả các hàm f : R+ −→ R+ thỏa mãn

f (x + y)2 = f (x)2 + 2 f (x y) + f ( y)2 , với mọi x, y > 0.

Bài toán 16 (Mohammad Jafari). Tìm hàm số f : R+ −→ R+ thỏa mãn

f ( f (x) + 2 y) = f (2x + y) + 2 y, với mọi x, y > 0.

Tập san Toán học STAR EDUCATION


30 TRẦN BÁ ĐẠT

Bài toán 17 (Mohammad Jafari). Cho hai hàm số f , g : R+ −→ R+ thỏa mãn

f (g(x) + y) = f (x) + g( y), với mọi x, y ∈ R+ ,


g( f (x) + y) = g(x) + f ( y), với mọi x, y ∈ R+ ,
f (1) = g(1).

Chứng minh rằng f (x) = g(x) với mọi x > 0.

Bài toán 18. Tìm tất cả các hàm f : R+ −→ R+ thỏa mãn

f (x + f ( y)) = 3 f (x) − 2x + f ( y), với mọi x, y > 0.

Bài toán 19. Tìm tất cả các hàm f : R+ −→ R+ thỏa mãn

f (x + f (x) + y) = f ( y) + 2x, với mọi x, y > 0.

Bài toán 20. Tìm tất cả các hàm f : R+ −→ R+ thỏa mãn

f (x + f (x) + 2 y) = f (2x) + y + f ( y), với mọi x, y > 0.

Bài toán 21. Tìm tất cả các hàm f : R+ −→ R+ thỏa mãn

f (x + f ( y)) + x = f (x + f (x)) + y, với mọi x, y > 0.

Bài toán 22 (IMO Shortlist 2005). Tìm tất cả các hàm f : R+ −→ R+ thỏa mãn

f (x) f ( y) = 2 f (x + y f (x)), với mọi x, y > 0.

Bài toán 23 (Thái Lan 2019). Tìm tất cả các hàm f : R+ −→ R+ thỏa mãn

f (x + y f (x) + y 2 ) = f (x) + 2 y, với mọi x, y > 0.

Bài toán 24 (Nhật Bản 2019). Tìm tất cả các hàm liên tục f : R+ −→ R+ thỏa

f ( y)
 ‹  y 
f + 1 = f x + + 1 − f (x), với mọi x, y > 0.
f (x) x

Tập san Toán học STAR EDUCATION


TRẦN BÁ ĐẠT

Bài toán 25. Tìm tất cả các hàm số f : R+ −→ R+ thỏa mãn

f (x + f (x + y)) = f (2x) + f ( y), với mọi x, y > 0.

Bài toán 26. Tìm tất cả các hàm f : R+ −→ R+ thỏa mãn

f (x + f (x) + 2 y) = f (2x) + 2 f ( y), với mọi x, y > 0.

Tập san Toán học STAR EDUCATION


Tài liệu tham khảo

[1] T. Andreescu, I. Boreico: Functional Equations. Electronic Edition (2007)

[2] P-C. Wu, : Functional Equation, Lecture Notes (2019)

[3] M. Jafari: 220 Problems Proposed by Mohammad Jafari.

[4] M. Jafari: 116 Algebra Problems Proposed by Mohammad Jafari.

32
Định lý Wolstenholme và ứng dụng

Kiều Đình Minh


(GV Trường THPT CHuyên Hùng Vương, Phú Thọ)
Lê Phúc Lữ
(Cao học ĐH KHTN TP Hồ Chí Minh)

1. Kiến thức cần nhớ

Định lý 1. Cho p là số nguyên tố, khi đó, ta có 2 định lý quan trọng:


(1) Wilson: (p − 1)! + 1 ≡ 0 (mod p).
(2) Fermat nhỏ: a p−1 ≡ 1 (mod p) với (a, p) = 1.

Các định lý này có thể chứng minh dựa vào tính chất của hệ thặng dư đầy đủ (TDĐĐ),
thu gọn và nghịch đảo của một số nguyên theo modulo p, cụ thể là:

Định lý 2. Với mọi số nguyên i ∈ {1, 2, 3, ..., p − 1} thì tồn tại duy nhất số j ∈
{1, 2, 3, ..., p − 1} sao cho i j ≡ 1 (mod p).

(nếu (a, m) = 1 thì ax + y sẽ chạy qua 1 hệ TDĐĐ của m khi x chạy qua 1 hệ TDĐĐ
của m). Đặc biệt: i = 1 ⇒ j = 1 và i = p − 1 ⇒ j = p − 1.

Định lý 3. Định lý Viète: Cho đa thức P(x) = an x n + an−1 x n−1 + ... + a2 x 2 + a1 x + a0


và giả sử đa thức này có n nghiệm là r1 , r2 , ..., rn khác 0. Khi đó, ta có:

an−1 (−1)n a0
r1 + r2 + ... + rn = − , r1 r2 ...rn = ,
an an
n
(−1)n−1 a1 X Y 1 1 1
 ‹
= r j = r1 r2 ...rn + + ... +
an i=1 j6=i
r1 r2 rn

Ta đã biết rằng một đa thức P(x) bậc n thì có tối đa n nghiệm. Điều này vẫn đúng
trong phương trình đồng dư, tức là P(x) ≡ 0 (mod m) với P(x) có bậc m cũng có tối
đa m nghiệm (không có cặp nghiệm nào đồng dư với nhau theo mod m). Chẳng hạn
như: P(x) = x 3 + x + 1 ≡ 0 (mod 3) có nghiệm là x ≡ 1 (mod 3).

Định nghĩa 1. Đồng dư thức của phân số: Ta quy ước ký hiệu ab ≡ dc (mod m), trong
đó a, b, c, d ∈ Z và b, d 6= 0, (b, m) = (d, m) = 1 khi ad − bc ≡ 0 (mod m). Chẳng
hạn, thay vì viết i j ≡ 1 (mod p) như ở trên, ta có thể viết 1i ≡ j (mod p).

33
34 KIỀU ĐÌNH MINH - LÊ PHÚC LỮ

Định lý 4. Đa thức Lagrange. Xét đa thức f (x) = (x + 1)(x + 2)...(x + p − 1) với p là


số nguyên tố. Khai triển ra, ta có: f (x) = x p−1 + a p−2 x p−2 + ... + a2 x 2 + a1 x + (p − 1)!
Định lý Lagrange về đa thức này nói rằng các hệ số ai với i = 1, 2, ..., p − 2 đều chia
hết cho p.

Cách 1. Thật vậy, Xét đa thức P(x) = f (x) − x p−1 − 1 , có bậc là p − 2. Hơn nữa
P(1) ≡ P(2) ≡ ... ≡ P(p − 1) ≡ 0 (mod p) nên đa thức này có p − 1 nghiệm, tức là tất
cả các hệ số của đa thức này phải chia hết cho p. Suy ra a p−2 , a p−3 , ..., a2 , a1 , (p−1)!+1
đều phải chia hết cho p. Định lý được chứng minh.
Cách 2. Nếu p là một số nguyên tố và 0 < i < p thì C pi chia hết cho p. Thật vậy,
p i−1
ta có C pi = i C p−1 . Nhưng (p, i) = 1, ∀i, 0 < i < p và C pi là một số nguyên nên
suy ra C pi chia hết cho p. Ta có f (x + 1) = (x + 2) ... (x + p), suy ra (x + p) f (x) =
(x + 1) f (x + 1) ⇒ p f (x) = (x + 1) f (x + 1) − x f (x) . Do đó

p x p−1 + pa1 x p−2 + ... + pa p−2 x + p! =


= (x + 1) p − x p + a1 (x + 1) p−1 − x p−1 + ... + a p−2 (x + 1)2 − x 2 + (x + 1 − x) (p − 1)!
 

Đồng nhất các hệ số của x p−2 , x p−3 , ..., x, ta được

pa1 = C p2 + C p−1
1
a1
pa2 = C p3 + C p−1
2
a1 + C p−2
1
a2
....
p−2
pa p−2 = C pp−1 + C p−1 a1 + ... + C21 a p−2

Áp dụng tính chất đã nêu trên, từ đẳng thức thứ nhất có p|a1 . Tương tự p|a1 , a2 , ..., a p−2 .

2. Định lý Wolstenholme

Định lý 5. Dạng 1. Cho p là số nguyên tố lớn hơn 3, khi đó ta có:

1 1 1 1
+ 2 + 2 + ... + ≡ 0(modp).
12 2 3 (p − 1)2

Chứng minh. Sử dụng nghịch đảo  2của2 một số theo modulo p thì tổng trên cũng là
2
tổng của một hoán vị của các số 1 , 2 , ..., (p − 1) và ta có

1 1 1 1 2 p(p − 1)(2p − 1)
+ + +...+ ≡ 12
+22
+...+(p − 1) = ≡ 0 (mod p).
12 22 32 (p − 1)2 6

Tập san Toán học STAR EDUCATION


KIỀU ĐÌNH MINH - LÊ PHÚC LỮ

Định lý 6. Dạng 2. Cho p là số nguyên tố lớn hơn 3, khi đó ta có:

1 1 1
+ + ... + ≡ 0 (mod p2 ))
1 2 p−1

Chứng minh. Dễ thấy rằng tổng trên đồng dư với 0 theo modulo p, định lý này đòi
hỏi một kết quả mạnh hơn. Chú ý rằng

1 1 1 1 1 1
 ‹
+ + ... + ≡ 1 · 2 · 3...(p − 1) + + ... + (modp2 )
1 2 p−1 1 2 p−1

nên sử dụng lại đa thức Lagrange và định lý Viète, ta cần chứng minh a1 ≡ 0( mod p2 ).
Thật vậy, từ đồng nhất thức:

(x + 1)(x + 2)...(x + p − 1) = f (x) = x p−1 + a p−2 x p−2 + ... + a2 x 2 + a1 x + (p − 1)!,

ta cho x = −p thì vế trái chính là (−1)(−2)...(−p + 1) = (−1) p−1 (p − 1)! = (p − 1)!.


Do đó: (−p) p−1 + a p−2 (−p) p−2 + ... + a2 (−p)2 = pa1 . Dễ thấy vế trái chia hết cho p3
nên vế phải cũng phải chia hết cho p3 hay p2 a1 .

Định lý 7. Dạng 3. Cho p là số nguyên tố lớn hơn 3, chứng minh rằng


p p−1
C2p ≡ 2 (mod p3 ) hay C2p−1 ≡ 1 (mod p3 ).

p (2p)! (2p−1)! p−1


Chứng minh. Hai kết quả trên là tương đương vì C2p = p!p! = 2 p!(p−1)! = 2C2p−1 .
p
Nếu áp dụng đồng nhất thức Vandermonde, ta chỉ chứng minh được rằng C2p ≡
2(modp2 ). Thật vậy, ta đã biết rằng:
p 2 2
C2p = (C p0 ) + (C p1 ) + ... + (C pp )2 .

Hai số hạng đầu và cuối bằng 1, còn các số hạng ở giữa đều chia hết cho p2 vì
p!
C pi = i!(p−i)! chia hết cho p với mọi i = 1, 2, 3, ..., p − 1. Để đưa từ modulo p2 sang p3 ,
ta phải có một cách hiệu quả hơn.

p (2p − 1)! (p + 1)(p + 2)...(p + p − 1)


C2p−1 − 1 = −1= −1
p!(p − 1)! (p − 1)!
(p + 1)(p + 2)...(p + p − 1) − (p − 1)!
=
(p − 1)!

Do p3 , (p − 1)! = 1 nên ta chỉ cần chứng minh (p + 1)(p + 2)...(p + p − 1) ≡ (p −
1)!(modp3 ) hay f (p) ≡ (p − 1)!(modp3 ). Tuy nhiên, f (p) − (p − 1)! = a p−2 p p−2 +
a p−3 p p−3 + ... + a2 p2 + a1 p chia hết cho p3 nên định lý được chứng minh.

3. Các ví dụ áp dụng

Tập san Toán học STAR EDUCATION


36 KIỀU ĐÌNH MINH - LÊ PHÚC LỮ

Bài toán 1. Cho p > 3 là số nguyên tố lẻ.

1. Chứng minh rằng: 1 − 212 + 312 − ... − (p−1)


1
2 ≡ 0 (mod p).

2. 2) Đặt 11 + 12 + ... + 1p = m
n; m, n ∈ Z và (m, n) = 1. Chứng minh rằng p4 |mp − n.

p2 −2pi
Lời giải. 1) Ta chỉ cần ghép các số hạng thích hợp: i12 − (p−i)
1
2 =
i 2 (p−i)2
chia hết cho p
với 1 ≤ i ≤ p − 1.
2) Ta có
1 1 1 m 1 mp − n
+ + ... + = − = .
1 2 p−1 n p np
mp−n
Áp dụng định lý Wolstenholme 2 cho vế trái thì np ≡ 0(modp2 ). Vì n chia hết cho
p nên np chia hết cho p2 , suy ra mp − n chia hết cho p4 .

Bài toán 2. (Mở rộng định lý Wolstenholme 3) Cho p > 3 là số nguyên tố và m, k ∈


Z+ . Chứng minh Cmpk −1 ≡ 1 (modp k+2 ).
p−1

Lời giải. Biến đổi trực tiếp, ta có đẳng thức f (−x) = f (x − p). Ta có:

p−1 (mp k − 1)! (mp k − p + 1)(mp k − p + 2)...(mp k − p + p − 1)


Cmpk −1 − 1 = − 1 = −1
(p − 1)!(mp k − p)! (p − 1)!
f (mp k − p) f (−mp k ) f (−mp k ) − (p − 1)!
= −1= −1=
(p − 1)! (p − 1)! (p − 1)!
p−1 p−2 2
Biểu thức trên tử khi khai triển ra có dạng (−mp k ) +a p−2 (−mp k ) +...+a2 (−mp k ) +
a1 (−mp k ). Vì p2 a2 nên dễ thấy rằng biểu thức trên chia hết cho p k+2 .
Ta thấy rằng định lý đã nêu rất mạnh nhưng cách chứng minh, biến đổi cũng tương
tự như định lý ban đầu.

Bài toán 3. (APMO 2006) Cho p > 3 là số nguyên tố và r là số cách đặt p quân cờ
lên bàn cờ hình vuông có kích thước p × p sao cho không có 2 quân cờ nào cùng hàng
(nhưng có thể cùng cột). Chứng minh rằng p5 |r.
p
Lời giải. Số cách chọn p quân cờ tùy ý là C p2 . Số cách chọn thỏa mãn đề bài là (trừ đi
p p
p cách không thỏa mãn) là: C p2 − p. Ta cần chứng minh C p2 − p ≡ 0(modp5 ), mà

p (p2 )! p2 · (p2 − 1)! p−1


C p2 = = = pC p2 −1
2
p!(p − p)! p · (p − 1)!(p − p)!
2

p−1
nên ta đưa về C p2 −1 ≡ 1(modp4 ). Áp dụng định lý Wolstenholme mở rộng, ta có
đpcm.

Tập san Toán học STAR EDUCATION


KIỀU ĐÌNH MINH - LÊ PHÚC LỮ

Bài toán 4. (Putnam 1980) Cho p là số nguyên tố, chứng minh rằng
p
X
C pi C p+i
i
≡ 2 p + 1 (modp2 ).
i=0

Lời giải. Với i = 1, 2, 3, ..., p − 1 thì C pi chia hết cho p và

(p + i)! (p + 1)(p + 2)...(p + i) (p + 1)(p + 2)...(p + i) − i!


i
C p+i −1= −1= −1=
i!p! i! i!

Dễ thấy tử số của biểu thức cuối chia hết cho p nên C p+i i
≡ 1(modp2 ). Với i = 0 thì
p
C p0 C p0 = 1 và với i = p thì C pp C2p ≡ 2(modp2 ) theo định lý Wolstenholme 3.
Nhận xét. Nếu x ≡ 0, y ≡ 1 (mod p) thì x y − x = x( y − 1) chia hết cho p2 nên
x y ≡ x(modp2 ). Từ đó suy ra
p
X p−1
X p−1
X p
X
C pi C p+i
i
≡1+ C pi C p+i
i
+2≡1+ C pi +2= C pi + 1 = 2 p + 1(modp2 ).
i=0 i=1 i=1 i=0

Bài toán 5. (KHTN 2010) Cho dãy số (un ) xác định bởi công thức:

u1 = 0,

(4n + 2)n3 3n + 1
un+1 = 4
un + ,n ≥ 1
(n + 1) (n + 1)4

Chứng minh rằng dãy số này chứa vô số số hạng nguyên dương.

Lời giải. Ta có: (n + 1)4 un+1 = (4n + 2)n3 un + 3n + 1, đặt vn = n3 un thì

(n + 1)vn+1 = (4n + 2)vn + 3n + 1 ⇔ (n + 1)(vn+1 + 1) = (4n + 2)(vn + 1).


4n+2
Lại đặt w n = vn + 1 thì w n+1 = n+1 vn . Ta cũng tính được v1 = 0, w1 = 1 nên

4n − 2 4n − 2 4n − 6 2n−1 (2n − 1)!! 2n−1 (2n − 1)!!


wn = w n−1 = · w n−2 = ... = w1 = ,
n n n−1 n! n!

trong đó (2n + 1)!! là tích các số lẻ từ 1 đến 2n + 1, (2n)! là tích các số chẵn từ 2 đến
2n. Ta biến đổi tiếp:

2n−1 (2n − 1)!! 2n−1 (n − 1)!(2n − 1)!! (2n − 2)!!(2n − 1)!! (2n − 1)!
= = = = C2n−1
n−1
.
n! n!(n − 1)! n!(n − 1)! n!(n − 1)!
C n−1 −1
Suy ra w n = C2n−1
n−1
⇒ vn = C2n−1
n−1
− 1 ⇒ un = 2n−1
n3 . Theo định lý Wolstenholme 3 thì
u p ∈ Z, mà có vô số số nguyên tố nên cũng có vô số số hạng của dãy là số nguyên. Ta
có đpcm.

Tập san Toán học STAR EDUCATION


38 KIỀU ĐÌNH MINH - LÊ PHÚC LỮ

1
Bài toán 6. 1) (IMO 1979) Chứng minh rằng 1 − 21 + 13 − ... + 1319
1
≡ 0 (mod1979).
2) Tổng quát hóa bài toán.

Lời giải. 1) Chú ý rằng 1979 nguyên tố, ta chỉ cần tìm cách tách ghép thích hợp.
1 1 1 1 1 1 1 1 1 1 1
 ‹
− + − ... + = + + + ... + −2 + + ... +
1 2 3 1319 1 2 3 1319 2 4 1318
1 1 1 1 1 1 1 1
 ‹
= + + + ... + − + + + ... +
1 2 3 1319 1 2 3 659
1 1 1 1 1 1 1 1
 ‹
≡ + + + ... + + + + + ... +
1 2 3 1319 1979 − 1 1979 − 2 1979 − 3 1979 − 659
989 
1 1 1 1 1 1 1 1
X ‹
= + + + ... + + + ... + ≡ + ≡ 0 (mod1979)
1 2 3 1319 1320 1978 i=1
i 1979 − i

2) Với ý tưởng tách ghép như trên, ta thấy rằng: - Nếu p = 3k + 2 là số nguyên tố thì
1 1 1 1
1 − 2 + 3 − ... + 2k+1 ≡ 0 (mod p).
- Nếu p = 3k + 1 là số nguyên tố thì 11 − 12 + 13 − ... − 2k
1
≡ 0 (mod p).

 2p  k
Bài toán 7. (Putnam 1996): Cho p ≥ 5 là số nguyên tố và đặt k =
P
3 thì C pi ≡
i=1
1 (modp2 ).

(−1)k−1
Lời giải. Với p là số nguyên tố, ta có bổ đề quan trọng sau: 1p C pk ≡ k (mod p) với
1 ≤ k ≤ p − 1.Chỉ cần biến đổi trực tiếp là có đpcm. Chú ý rằng đây là một ý tưởng
quan trọng, sử dụng cho nhiều bài khác.
p
Trở lại bài toán, ta thấy rằng C pk ≡ (−1)k−1 k (modp2 ). Đến đây, áp dụng bài toán 5 ở
trên với kết quả: tổng đan dấu của đồng dư với 0 theo modulo p, ta có đpcm.

a2
Bài toán 8. (IMO Shortlist 2013) Cho p là số nguyên tố lẻ. Đặt Sa = 1a + 2 + ... +
a p−1 + +
p−1 , a ∈ Z Đặt S3 + S4 − 3S2 = n với m, n ∈ Z . Chứng minh rằng p| m.
m

k k
Lời giải. Áp dụng bổ đề ở bài trên, ta có: ak ≡ (−1)k−1 ap C pk (mod p). Suy ra
p−1 p
‚ Œ
X a k
1 X (a − 1) p − a p + 1
Sa =≡ (−1)k−1 C pk = ap − 1 − (−1)k C pk a k = (mod p).
k=1
p p k=0
p
Do đó:
2 p − 3 p + 1 + 3 p − 4 p + 1 − 3(2 − 2 p )
S3 + S4 − 3S2 ≡
p
4p − 4 · 2p + 4 (2 p − 2)2
=− =− (mod p)
p p
(2 p −2)2
Theo định lý Fermat nhỏ thì 2 p ≡ 2 (mod p) nên p2 (2 p − 2)2 hay p ≡ 0 (mod p)
Từ đó suy ra đpcm. Đây là bài toán N7 trong đề IMO Shortlist 2011. Ý tưởng khó chính
là phát hiện và sử dụng được bổ đề trên.

Tập san Toán học STAR EDUCATION


KIỀU ĐÌNH MINH - LÊ PHÚC LỮ

Bài toán 9. (Chọn đội tuyển Nghệ An 2012) Cho p > 3 là số nguyên tố và đặt
M = {1, 2, 3, ..., p}. Với mỗi số nguyên dương k thỏa 1 ≤ k ≤ p, xét tập hợp Ek =
p−1
{A ⊂ M : |A| = k} và đại lượngx k = (min A + max A). Chứng minh rằng
P P
x k C pk ≡
A∈Ek k=1
0 (modp3 ).

Lời giải. Trước hết, với mỗi phần tử i, ta đếm có bao nhiêu tập hợp có k phần tử nhận
i là min. Nếu i = 1 thì có C p−1
k−1
tập hợp; nếu i = 2 thì có C p−2
k−1
tập hợp. Từ đó suy ra,
k−1
có tất cả C p−i tập hợp nhận i là phần tử nhỏ nhất, trong đó p − i ≥ k − 1. Tương tự,
k−1
có tất cả Ci−1 tập hợp nhận i là phần tử lớn nhất, trong đó i ≥ k. Do đó:
X X X
xk = (min A + max A) = k−1
iC p−i + k−1
iCi−1
A∈Ek p−i+1≥k i≥k
X X
= (p + 1 − k−1
i)Ci−1 + k−1
iCi−1
i≥k i≥k
X
= (p + 1) k−1
Ci−1 = (p + 1)C pk
i≥k

p−1 p−1 € Š
p
x k C pk ≡ (p + 1) C pk C pk = (p + 1) C2p − 2 ≡ 0(modp3 ) theo định
P P
Từ đây suy ra:
k=1 k=1
lý Wolstenholme 3.

Bài toán 10. (J28.Mathematical


 2p  Reflections) Cho p là số nguyên tố sao cho p ≡
1 1 1
1 (mod3) và đặt q = 3 . Nếu 1.2 + 3.4 + ... + (q−1)q = mn với m, n ∈ Z, thì p|m.

 2p 
Lời giải. Đặt p = 3k + 1 và q = 3 = 2k. Khi xét phương trình modulo p ta
1 1 1
phải chứng minh nó đồng dư với 0 mod p. Ta có S = 1.2 + 3.4 + ... + (q−1)q = 11 −
1 1
2 + − 14 + ... + q−1
3
1
− 1q . Bây giờ ta nhóm các phân số lại và thay q = 2k , được
q q/2 2k k
1
P 1 1 1 1 1 1
S = = i . Từ định lý Wolstenholme, ta có 1 + 2 + ... + p−1 ≡
P P P
i − 2 2i i −
i=1  i=1 i=1 i=1
0 modp . Bởi vì −i ≡ p − i (modp) , nên suy ra
2

p−1 p−1 k 3k k
X 1 X 1 X 1 X 1 X 1
S= − + ≡0− + ≡ 0 (modp) .
i=1
i i=2k+1
i i=1
p − i i=2k+1
i i=1
3k + 1 − i

Bài toán 11. (O.264. Mathematical Reflections) Cho p > 3 là một số nguyên tố.
Chứng minh rằng
(p−1)/2
1 1 1
 X  ‹
2 p−1
≡ 1 modp 2
⇔ 1 + + ... + ≡ 0 (mod p).
k=2
k 2 k−1

Tập san Toán học STAR EDUCATION


40 KIỀU ĐÌNH MINH - LÊ PHÚC LỮ

Lời giải. Ta cần chỉ ra rằng


(p−1)/2
X H k−1 (1) 1 € 2 Š
= H(p−1)/2 (1) − H(p−1)/2 (2) ≡ 2q2 (modp) .
k=2
k 2
1 1
Với H k−1 (d) = 1 + 2d
+ ... + (k−1)d
. Từ đó, theo định lý Wolstenholme H p−1 (1) ≡
H p−1 (2) ≡ 0 (modp) với số nguyên tố p > 3, suy ra
p−1   p−1
2p − 2 1 X p X (−1)k−1
2q = = ≡ (modp) =
p p k=1 k k=1
k
p−1 (p−1)/2
X 1 X 1
= −2 = H p−1 (1) − H(p−1)/2 (1) ≡ −H(p−1)/2 (1) (modp)
k=1
k k=1
2k

(p−1)/2 p−1
1 X 1 1 X 1 1
H(p−1)/2 (2) = + 2
≡ H p−1 (2) ≡ 0 (modp)
2 k=1 k 2 2 p+1 (p − k) 2
k= 2

Vì vậy
1€ 2 Š 1
(−2q)2 + 0 ≡ 2q2 (modp) .

H(p−1)/2 (1) − H(p−1)/2 (2) ≡
2 2

Bài toán 12. (O.354. Mathematical Reflections) Tìm tất cả các số nguyên tố p sao cho
biểu thức
p2
1 + 21 + ... + 1
p−1

là một số nguyên.

Lời giải. Dễ kiểm tra rằng biểu thức là một số nguyên với p = 2, 3, 5, 7. Ta chỉ ra rằng
chỉ có những số nguyên tố như vậy thỏa mãn bài toán. Xét số nguyên tố p ≥ 11. Đặt
p2
H p−1 = 1 + 12 + ... + p−1
1
= ab , (a, b) = 1. Theo định lý Wolstenholme có p2 |a. Vì vậy H p−1
là một số nguyên khi a = p2 . Ta xét các trường hợp sau
112
1. Nếu p = 11 thì 23 .32 .7 chia hết b. Vậy 1 < H10 ≤ 8.9.7 < 1, mâu thuẫn.
p+1
2. Nếu 13 ≤ p ≤ 53 thì có hai số nguyên tố q1 , q2 sao cho 2 ≤ q1 < q2 < p − 1.
p2 p2
Do 2 q1 q2 |b, ta có 1 < H p−1 ≤
3
8q1 q2 < 2(p+1)2
< 1, mâu thuẫn.

3. Cuối cùng, nếu p ≥ 59 thì tồn tại m ≥ 5 sao cho 2m ≤ p − 1 < 2m+1 . Theo định
p+1
lý Chebyshev (bổ đề Bertrand), tồn tại số nguyên tố q sao cho 2 ≤ q < p − 1.
Do 2m q|b, ta có

p2 2m+1 + 1 p 1
4, 07 < log2 59 < H p−1 ≤ m < m−1 <4+ < 4, 07.
2 q 2 (p + 1) 16

Điều mâu thuẫn này hoàn tất chứng minh.


Nhận xét. Đây là bài toán khó, có sự kết hợp nhiều kết quả quan trọng. Về bổ đề
Bertrand bạn đọc có thể xem ở số 432 tháng 6 năm 2013 trên tạp chí TH và TT.

Tập san Toán học STAR EDUCATION


KIỀU ĐÌNH MINH - LÊ PHÚC LỮ

4. Bài tập tự luyện


1
1. Đặt 12016 1
+ 22016 + ... + 201612016 = a
b với a, b ∈ Z+ và (a, b) = 1. Chứng minh rằng
tổng a + b chia hết cho 2017.

2. Giả sử p là số nguyên tố lớn hơn 3; q = 3 ; m, n ∈ Z+ thỏa mãn


 2p  m
n = 1 − 12 +
1 1 q−1 1
3 − 4 + ... + (−1) q . Chứng minh rằng p|m.

3. Cho p là số nguyên tố n là số nguyên dương sao cho n ≥ p. Chứng minh rằng


€ Š2
2p
n
Cn+p n
− Cn+2p − Cn+p chia hết cho p2 .

k
1
4. (O.292. Mathematical Reflections) Với mỗi số nguyên dương k, đặt Tk =
P
j2 j
.
j=1
p−2
Tk
≡ 0 (modp).
P
Tìm tất cả các số nguyên tố p sao cho k+1
k=1

5. (O.54. Mathematical Reflections) Cho p = 2q + 1 là một số nguyên tố lớn hơn


1
P
3. Chứng minh rằng p chia hết tử số của i j , ở đây tổng được lấy trên
1≤i, j≤q,i+ j>q
tất cả các cặp có thứ tự (i, j) .
p−1

 Với mọi số nguyên tố p ≥ 5, p = 2k + 1 thì (−1)


k
6. (Morley, 1895) 2
C2k ≡
p−1 3
4 modp .

Tập san Toán học STAR EDUCATION


Tài liệu tham khảo

[1] Tạp chí Mathematical Reflections.

[2] Diễn đàn Toán học mathscope.org.

[3] Tạp chí Toán học và tuổi trẻ.

[4] Diễn đàn artofproblemsolving.com.

42
Bổ đề ERIQ và ứng dụng

Trương Tuấn Nghĩa


(Lớp 11A1, Trường THPT Chuyên KHTN Hà Nội)

1. Giới thiệu
Bổ đề ERIQ được đặt tên bởi tác giả Kostas Vittas trên diễn đàn AoPS với nick name
vittasko. (là các chữ viết tắt của cụm từ Equal Rat ios I n Quad r ilat er al). Nội dung
bổ đề:
Cho tứ giác ABC D, lấy các điểm M , N nằm trên cạnh AD, BC sao cho MM DA = NN CB . Khi
đó, trung điểm của AB, M N , C D thẳng hàng.

Chứng minh. Gọi X , Y, Z là trung điểm của AB, M N , C D. Lấy P, Q nằm trên X M , X N
sao cho DP, CQ k AB.

Khi đó, theo định lý Thales, ta có MM DA = DP


AX
= M
M P ; N C = CQ = NQ . Suy ra DP = CQ;
X NB AY NX

M P = NQ hay M N k PQ. Do DP = CQ; DP k CQ nên P CQD là hình bình hành hay Z


MX NX

là trung điểm PQ.

Kết hợp với Y là trung điểm của M N , ta có X , Y, Z thẳng hàng.

Nhận xét. Ta có thể chứng minh X , Y, Z là các điểm chia cùng tỉ lệ trên AB, M N , C D
thẳng hàng bằng cách tương tự. Tiếp theo, ta sẽ đến với một số các mở rộng và ứng dụng
của bổ đề trên.

2. Ứng dụng

43
44 TRƯƠNG TUẤN NGHĨA

Bài toán 1. Cho tứ giác ABC D, lấy M , N nằm trên cạnh AD, BC sao cho MM DA = NN CB .
Lấy các điểm X , Y, Z sao cho các tam giác X AB, Y M N , Z C D đồng dạng và X , Y, Z lần
lượt nằm trên các nửa mặt phẳng bờ AB không chứa C, M N không chứa D và C D
chứa A. Chứng minh rằng X , Y, Z thẳng hàng.

Lời giải. Lấy P, Q ∈ X M , X N sao cho DP k X A, CQ k X B.

Theo định lý Thales, DP = X A. MM DA , CQ = X B. NN CB mà MM DA = NN CB nên DP = CQ Mặt


khác vì ∠AX B = ∠C Z D nên ∠Z DP = ∠Z CQ. Do đó, Í Z DP =Í Z CQ(c.g.c) dẫn tới
∠P Z D = ∠QZ C hay ∠C Z D = ∠P ZQ. Vì DP k X A, CQ k X B nên XMMP = XNQ N
(= MM DA ) nên
XY0 XY0
M N k PQ. Lấy Y 0 ∈ X Z sao cho Y 0Z = XM
MP = XN
NQ . Theo định lý Thales, Y 0Z = XM
MP = XN
NQ
nên
Y 0 M k Z P, Y 0 N k ZQ
XY0 XY0
Y 0 M = Y 0 N (= Z P. = ZQ. )
XZ XZ

Hay ∠M Y 0 N = ∠M Y N , Y 0 M = Y 0 N . Do đó, Y 0 ≡ Y hay X , Y, Z thẳng hàng.

Bài toán 2. Cho tứ giác ABC D có phân giác trong của các góc ∠A, ∠B, ∠C, ∠D đồng
quy tại I. AD cắt BC tại E, AB cắt C D tại F . Gọi M , N là trung điểm AC, E F. Chứng
minh rằng M , N , I thẳng hàng.

Lời giải. Gọi P, Q là giao điểm của đường thẳng qua I, vuông góc với I B với BA, AC.
Đầu tiên, dễ thấy I là giao 3 phân giác Í ABE. Do BI là phân giác ∠ABC nên Í BPQ
cân tại B hay I là trung điểm PQ.

Tập san Toán học STAR EDUCATION


TRƯƠNG TUẤN NGHĨA

Ta có ∠BPQ = 90◦ − ∠ABE 2 = ∠AEB


2 + ∠BAE
2 , ∠IAB =
∠BAE
2 nên ∠P IA = ∠AEB2 . Tương tự
thì ∠E IQ = 2 . Do đó, Í P IA ∼Í QEA(g.g) nên PA.QE = P I.QI. Hoàn toàn tương
∠BAE

tự, P F.QC = P I.QI. Vậy ta có FA


PA
= QC
QE nên theo bổ đề ERIQ, M , I, N lần lượt là trung
điểm của PQ, AC, E F thẳng hàng.

Bài toán 3. Cho tứ giác ABC D nội tiếp, không là hình thang. Gọi E, F là giao điểm của
các cặp đường thẳng (AB, C D); (AD, BC). Giả sử phân giác của góc ∠AEC, ∠AF B cắt
nhau tại I. Gọi M , N lần lượt là trung điểm của AC, BD. Chứng minh rằng I ∈ M N .

Lời giải. Giả sử các điểm có vị trí như hình vẽ, các trường hợp khác tương tự.

Tập san Toán học STAR EDUCATION


46 TRƯƠNG TUẤN NGHĨA

Gọi P, Q lần lượt là giao điểm của F I với AB, C D. Do ∠ABC + ∠C DA = 180◦ nên
∠FAB = ∠F C D nên Í FAB ∼Í F C D(g.g)(∗) và

∠E PQ = ∠FAB + ∠AF I = ∠F C D + ∠BF I = ∠EQP

hay Í E PQ cân tại E. Mà E I là phân giác ∠AE D nên I là trung điểm PQ. Mặt khác
CQ
theo (*), FFAB = FF CD nên theo tính chất đường phân giác, AP
P B = QD . Do đó theo bổ đề
ERIQ, trung điểm AC, BD, PQ thằng hàng hay I ∈ M N . (đpcm)

Bài toán 4. (AOPS). Cho Í ABC, trực tâm H,P bất kỳ trên BC, X bất kỳ trên H P.
Gọi E, F 6= A là giao điểm của đường tròn đường kính AX với CA, AB. Tiếp tuyến tại
E, F của (AE F ) cắt nhau tại T . Đường thẳng qua P vuông góc BC cắt CA, AB tại Z, Y .
Gọi L là trung điểm Z Y . Chứng minh rằng LT chia đôi BC.

Lời giải. Trước hết, ta phát biểu và chứng minh hai bổ đề sau:
Bổ đề 1. Cho Í ABC, đường cao BE, C F . Gọi M là trung điểm của BC. Khi đó, M E, M F
là tiếp tuyến của (AE F ).
Bổ đề trên có thể chứng minh dễ dàng qua các phép cộng góc.
Bổ đề 2. Cho tứ giác ABC D, AB cắt C D tại E. Gọi H, K là trực tâm của Í EAD, Í EBC.
Khi đó, H K là trục đẳng phương của 2 đường tròn đường kính BD, AC.

Chứng minh. Gọi M , N là hình chiếu của B, C lên EC, EB. Khi đó, M N BC là tứ giác
nội tiếp nên K N .KC = K M .K B.

Mặt khác, M , N lần lượt nằm trên đường tròn đường kính BD, AC mà K N .KC =
K M .K B nên K nằm trên trục đẳng phương của 2 đường tròn trên. Chứng minh tương
tự, H K là trục đẳng phương của đường tròn đường kính BD và đường tròn đường
kính AC.

Trở lại bài toán,

Tập san Toán học STAR EDUCATION


TRƯƠNG TUẤN NGHĨA

Gọi M , N là giao điểm của X F, X E với CA, AB. Khi đó, theo bổ đề 1 dễ có T là trung
điểm của M N nên theo bổ đề ERIQ, ta chỉ cần chứng minh BN BZ = C Y . Gọi U, V là hình
CM

chiếu của N , M lên BC. Theo bổ đề 2 thì H X là trục đẳng phương của đường tròn
đường kính M B, N C. Dễ thấy U, V lần lượt nằm trên đường tròn đường kính C N , BM
nên và P nằm trên H X , BC nên ta có PU.P C = P V.P B hay PU PB
= PP VC , và theo định lý
Thales thì BZ = C Y . Vậy ta thu được LT chia đôi BC.
BN CM

Bài toán 5. Cho Í ABC, P bất kỳ trên BC, J là trung điểm của AP. Gọi E, F là giao
điểm của (J, JA) với CA, AB. Gọi L là tâm đường tròn ngoại tiếp Í J E F . Chứng minh
rằng khi P di chuyển trên BC thì L chuyển động trên đường thẳng cố định.

Lời giải. Trước hết ta chứng minh bổ đề sau: Cho Í ABC, lấy điểm M cố định trên
BC, P bất kỳ trên BC. Gọi E, F là hình chiếu của P lên CA, AB, K, L là hình chiếu của
M lên CA, AB. Khi đó, tỉ số EK
F L không phụ thuộc vào vị trí của P trên BC.

Tập san Toán học STAR EDUCATION


48 TRƯƠNG TUẤN NGHĨA

Chứng minh. Gọi X , Y là hình chiếu của M , P lên P F, M K. Khi đó,

M X = LF = M P. cos ∠X M P = M P.cos∠ABC;
Y P = K E = M P. cos ∠Y P M = M P. cos ∠AC B.

cos ∠AC B
Do đó, EK
FL = cos ∠ABC .

Trở lại bài toán,

Lấy M , N cố định trên BC. X , Z là hình chiếu của M lên AB, AC; Y, T là hình chiếu
của N lên AB, AC. Khi đó, theo bổ đề 1 thì dễ có được YY XF = TT ZE . (1) Do J là tâm đường
tròn ngoại tiếp Í AE F nên ∠F J E = 2.∠BAC. Mà L là tâm đường tròn ngoại tiếp của
Í J E F nên ∠F LE = 360◦ − 4.∠BAC. Theo (1) và bổ đề ERIQ thì các đỉnh của tam
giác cân có đáy F E, Y T, X Z và có góc ở đỉnh là 360◦ − 4.∠BAC thì thẳng hàng mà
M , N cố định nên L nằm trên đường thẳng cố định. (đpcm)

Bài toán 6. (Nguyễn Văn Linh) Cho Í ABC, đường cao AD, K ∈ AD. Gọi E, F lần
lượt là giao điểm của BK, C K với CA, AB. Giả sử DE, DF cắt lại đường tròn ngoại tiếp
Í ABD; Í AC D tại M , N . Gọi T là trung điểm của M N . Chứng minh rằng AT chia đôi
đoạn thẳng E F.

Lời giải. Gọi BP, CQ là đường cao của Í ABC, đường thẳng qua A song song BC cắt
DE, DF tại K, L. Theo kết quả quen thuộc DF, DE đối xứng nhau qua AD và DQ, DP
đối xứng nhau qua AD. Nên A là trung điểm của K L.

Tập san Toán học STAR EDUCATION


TRƯƠNG TUẤN NGHĨA

Khi đó, theo bổ đề ERIQ, ta chỉ cần chứng minh NN FL = M K


M E . Ta có, A, M , P, D, Q nằm
trên đường tròn và A, N , Q, D, C nằm trên đường tròn. (1)
Do đó, ∠N AQ = ∠N DQ, ∠M AP = ∠M DP. Do DF, DE đối xứng nhau qua AD và
DQ, DP đối xứng nhau qua AD,nên ∠QDF = ∠P DE. Từ (1), ta cũng có ∠AQN =
∠ADN = ∠ADM = ∠AP M . Do đó, Í ANQ ∼Í AM P. (2)
sin L FA KA sin KAE
Mặt khác, FALL = sin LAF ; K E = sin K EA . Vì AK = AL; ∠FAL = ∠ABC; ∠EAK = ∠AC B, nên

F L KA sin LFA sin KAE FL


. = . =
AL K E sin FAL sin K EA K E
sin LFA sin KAE sin AC B sin LFA AB sin LFA
= . = . = . .
sin K EA sin FAL sin ABC sin K EA AC sin K EA

Ta lại có

sin LFA sin N FA sin M AP AN M E AN M E AQ M E AC M E


= . = . = . = . = . .
sin K EA sin N AF sin M EA F N M A AM F N AP F N AB F N

(do (2)). Vậy nên


FL AB sin LFA AB.AC M E ME
= . = . = .
KE AC sin K EA AC.AB N F NF

Bài toán 7. (Chọn đội tuyển PTNK TPHCM) Cho Í ABC, trực tâm H. Lấy điểm M
bất kỳ trên cung BH C của (BH C). Trên BM , C M lấy các điểm E, F sao cho ∠ECA =
∠F BA = 90◦ . Chứng minh rằng khi M chuyển động thì trung điểm E F luôn nằm trên
đường thẳng cố định.

Lời giải. Ở bài toán này, ta có hai hướng tiếp cận như sau:
Cách 1. Gọi N là giao điểm của C E, BF. Lấy P đối xứng với N qua BC, BP, C P lần lượt
cắt C E, BF tại X , Y. Dễ dàng chứng minh B, H, M , P, C nằm trên đường tròn.

Tập san Toán học STAR EDUCATION


50 TRƯƠNG TUẤN NGHĨA

Ta sẽ chứng minh YX FE không đổi khi M chuyển động trên cung BH C. Do ∠BM C =
∠BN C = 180◦ − ∠BAC nên ∠C M E = ∠C N F hay 4 điểm M , N , E, F nằm trên đường
tròn nên ∠C F Y = ∠BEX . (1) Mặt khác, do B, H, M , P, Cnằm trên đường tròn nên
∠Y C F = ∠M C P = ∠X BE. (2) Từ (1) và (2) suy ra Í C Y F ∼Í BX E(g.g). Do đó,
Y F = C Y không đổi. Vậy Y F không đổi khi M chuyển động trên cung BH C nên theo
XE BX XE

bổ đề ERIQ, trung điểm của E F luôn nằm trên đường thẳng cố định.
Cách 2. Trước hết ta phát biểu và chứng minh bổ đề sau: (IMO2009 Shortlist G4)
Cho tứ giác ABC D nội tiếp đường tròn (O). AC cắt BD ở E, AD cắt BC tại F. Gọi M , N
lần lượt là trung điểm của AB, C D. Khi đó, E F tiếp xúc với đường tròn ngoại tiếp của
Í EMN.

Chứng minh. Gọi I là trung điểm của E F. Xét tứ giác toàn phần AEBF.C D có I, M , N
lần lượt là trung điểm của các đường chéo E F, AB, C D nên I, M , N thẳng hàng.

Tập san Toán học STAR EDUCATION


TRƯƠNG TUẤN NGHĨA

Ta sẽ chứng minh I M .I N = I E 2 . Gọi L, P, T lần lượt là giao điểm của AB với C D,


E F với AB, C D. Khi đó, (LP, AB) = (LT, C D) = −1 nên áp dụng hệ thức M aclaur in
và ABC D là tứ giác nội tiếp, ta thu được LM .LP = LA.LB = LC.LD = LT .LN nên
4 điểm M , P, N , T nằm trên đường tròn. Do đó, I M .I N = I P.I T . Mặt khác, ta lại có
(E F, P T ) = −1 nên theo I E 2 = I T .I P. Vậy I M .I N = I E 2 . Do đó, E F là tiếp tuyến của
đường tròn ngoại tiếp Í E M N . (đpcm)

Trở lại bài toán,


Gọi N là giao điểm của C E, BF. Lấy I, P, Q lần lượt là trung điểm của BC, E F, M N .

Theo lời giải thứ nhất, ta có 4 điểm M , N , E, F nằm trên đường tròn nên theo bổ đề 4
2
thì BC là tiếp tuyến của (QC P) hay I C 2 = IQ.I P. Do đó, I II C : P ↔ Q. (1)
Mặt khác VN2 : Q 7→ M mà M chuyển động trên cung BH C nên Q chuyển động trên
đường tròn (ω) cố định. (2)
2
Từ (1) và (2), ta thu được P chuyển động trên đường thẳng ảnh của (ω) qua I II C :
P ↔ Q.

Nhận xét. Qua các bài toán trên, ta có thể thấy được ứng dụng của bổ đề ERIQ trong
các bài toán hình học. Sau đây sẽ là một số các bài toán luyện tập.

3. Luyện tập
1. Cho Í ABC nội tiếp (O). Tiếp tuyến của (O) tại A cắt tiếp tuyến của (O) tại B, C
lần lượt tại E, F . Gọi M , N là trung điểm của BF, C E. Đường thẳng qua O và
vuông góc với OA cắt BC tại S. Chứng minh rằng M N chia đôi SO.

2. Cho Í ABC, trực tâm H, trung tuyến AM . P bất kỳ trên H M . Đường tròn đường
kính AP cắt CA, AB tại E, F . Tiếp tuyến tại E, F của (AE F ) cắt nhau tại T . Chứng
minh rằng T B = T C.

Tập san Toán học STAR EDUCATION


52 TRƯƠNG TUẤN NGHĨA

3. Cho Í ABC, đường tròn (K) đi qua B, C cắt CA, AB tại E, F . Gọi H là giao điểm
của BE, C F. Lấy P bất kỳ trên BC. Đường thẳng qua P và song song với AH cắt
CA, AB tại X , Y.Lấy Q bất kỳ trên H P. Đường thẳng qua Q song song với BE, C F
cắt CA, AB tại X , Y, Z, T.
a) Chứng minh rằng 4 điểm X , Y, Z, T nằm trên đường tròn (L).
b) K L cắt trung trực PQ tại Z. Chứng minh rằng Í Z PQ ∼Í K BC.

4. Cho Í ABC, P bất kỳ trên BC. Đường thẳng qua P song song với CA, AB cắt
trung trực BA, AC tại M , N . Chứng minh rằng khi P chuyển động trên BC, tâm
đường tròn ngoại tiếp của Í M N P luôn nằm trên một đường thẳng cố định.

5. (Việt Nam TST 2008) Cho Í ABC nhọn không cân nội tiếp (O). Với k ∈ R+ ,
trên các đoạn phân giác AD, BE, C F, lấy M , N , P sao cho AM AD = BE = C F = k. Vẽ
BN CP

đường tròn (O1 ) đi qua A, M và tiếp xúc với OA; vẽ đường tròn (O2 ) đi qua B, N
và tiếp xúc với OB; vẽ đường tròn (O3 ) đi qua C, P và tiếp xúc với OC. Tìm tất
cả các giá trị k sao cho (O1 ), (O2 ), (O3 ) có đúng hai điểm chung.

6. Cho tam giác ABC nhọn không cân có điểm D thay đổi trong tam giác sao cho
∠ABD = ∠AC D, lấy E ∈ AB, F ∈ AC sao cho D là trực tâm tam giác AE F. Chứng
minh rằng:
a) Trung tuyến đỉnh D của tam giác DE F luôn đi qua điểm cố định.
b) Trung trực E F luôn đi qua điểm cố định.
c) Tâm đường tròn ngoại tiếp tam giác (DE F ) luôn thuộc đường cố định.
d) Trục đẳng phương của (BDE), (C DF ) luôn đi qua một điểm cố định.

Tập san Toán học STAR EDUCATION


Tài liệu tham khảo

[1] Nguyễn Văn Linh, Về bài 3 đề VMO 2016.

[2] Nguyễn Văn Linh, 2015, Định lý ERIQ, https://nguyenvanlinh.wordpress.


com/.

[3] Diễn đàn artofproblemsolving.com/community

[4] Trần Quang Hùng, Các bài giảng đội tuyển.

53
Đếm bằng hai cách trong tổ hợp

Nguyễn Tăng Vũ
(Giáo viên PTNK TP Hồ Chí Minh)

1. Giới thiệu
Đếm bằng hai cách là một phương pháp hay gặp trong đời sống, ví dụ bài toán sau:
Một công ty nhập vào 3 xe hàng A, B, C gồm hai loại hàng I và I I. Trong đó xe A có 3
loại I và 2 loại I I, xe B có 4 loại I và 6 loại I I, xe C có 4 loại I và 6 loại I I. Tính số
lượng hàng mà công ty nhâp vào.
Đây là bài toán khá đơn giản, để giải bài toán ta có thể lập bảng và khi đó ta có thể
tính bằng 2 cách như sau: Tính tổng số hàng trên mỗi xe rồi cộng lại; hoặc ta có thể
tính tổng số hàng loại I trên 3 xe,tổng số hàng loại 2 trên 3 xe, rồi sau đó cộng lại.

Trên đây là một ví dụ của tính bằng hai cách, ta có thể tính tổng theo dòng hoặc có
thể tính tổng theo cột. Tổng quát hơn ta có công thức đại số sau
‚ Œ ‚ Œ
X X X X X
ai j = ai j = ai j
i∈I, j∈J i∈I j∈J j∈J i∈I

Trong một số tình huống đề bài yêu cầu đếm số phần tử của một tập hợp mà không
quan tâm ta đếm bằng cách nào, khi đó đếm bằng hai cách cho ta cùng một đáp số
giống nhau, khi đó ta sẽ thiết lập được một đẳng thức tổ hợp. Một ví dụ đơn giản
như đếm số tập con của tập có n phần tử, ta có thể đếm số tập có k phần tử với
k = 0, 1, ..., n, lấy tổng ta được

Cn0 + Cn1 + .... + Cnn .

Nhưng nếu ta đếm bằng cách khác như sau: xét một tập hợp A bất kì, khi đó phần tử
i có thể thuộc A hoặc i không thuộc A, mỗi phần tử có 2 trường hợp, mà có n phần tử
nên số tập A là 2n . Từ đó ta có đẳng thức

Cn0 + Cn1 + .... + Cnn = 2n .

Đếm bằng hai cách cho ta một phương pháp để chứng minh đẳng thức liên quan tới
hệ số khai triển nhị phân hay các đẳng thức tổ hợp.

54
NGUYỄN TĂNG VŨ

Ngoài ra đếm bằng hai cách có thể áp dụng trong các bài toán bất đẳng thức, cực trị
tổ hợp hay một số bài toán chứng minh sự tồn tại. Để sử dụng phương pháp đếm bằng
hai cách, đòi hỏi học sinh phải biết và vận dụng tốt các phép đếm cơ bản. Bài viết này
được sử dụng để giảng dạy cho học sinh lớp 10 chuyên Toán, các em mới bước đầu
làm quen với các bài toán tổ hợp nói chung và các bài toán đếm nói riêng nên ví dụ
được nêu ra có độ khó không cao giúp các em làm quen với phương pháp này.

2. Chứng minh các đẳng thức tổ hợp

Bài toán 1. Cho các số nguyên dương n và k với 0 < k ≤ n. Chứng minh:

1. Cnk = Cn−1
k
+ Cn−1
k−1

k≥0 Cn = 2
P 2k n−1
2.

Lời giải

1. Dễ thấy vế trái của đẳng thức là số cách chọn k phần tử từ n phần tử. Để chọn
k phần tử từ n phần tử ta có thể làm như sau: Xét phần tử a, nếu a được chọn
k−1
thì ta cần chọn thêm k − 1 phần tử từ n − 1 phần tử còn lại ta có Cn−1 cách. Nếu
k
a không được chọn, ta chọn k phần tử từ n − 1 phần tử còn lại, ta có Cn−1 . Do
đó số cách chọn trong hai trường hợp là Cn−1 + Cn−1 . Từ đó ta có đpcm.
k k−1

2. Ta xét bài toán "đếm số cách chọn một số chẵn phần tử từ n phần tử": P
Cách 1: Ta có số cách chọn 2k phần tử từ n phần tử là Cn2k . Khi đó k≥0 Cn2k lần
tổng số cách chọn một số chẵn phần tử từ n phần tử.
Cách 2: Xét một phần tử a, thì có hai khả năng a được chọn hoặc a không được
chọn, ta có 2 trường hợp. Khi đó với n − 1 phần tử đầu tiên, thì số trường hợp là
2n−1 . Tới phần tử thứ n, nếu ta đã chọn được một số chẵn phần tử thì ta không
chọn, còn nếu ta đã chọn được một số lẻ phần tử thì phần tử này sẽ được chọn,
do đó số cách chọn là 2n−1 .

Bài toán 2. Cho các số nguyên dương n và k với 0 ≤ k ≤ n. Chứng minh rằng:

1. kCnk = nCn−1
k−1

Pn
2. k=0
kCnk = n2n−1

Tập san Toán học STAR EDUCATION


56 NGUYỄN TĂNG VŨ

Lời giải
1. Xét bài toán "Một đội văn nghệ có n thành viên, có bao nhiêu cách chọn k người
thể hiện một tiết mục hát tốp ca trong đó có một bạn hát sô lô".
Cách 1: Chọn đội văn nghệ gồm k người từ n ta có số cách là Cnk , từ k người này
ta chọn một người hát sô lô có k cách. Khi đó số cách chọn là kCnk .(1)
Cách 2: Chọn người hát sô lô trước, có n cách, sau đó chọn k − 1 người từ n − 1
k−1 k−1
người còn lại có Cn−1 cách. Vậy số cách chọn là nCn−1 . (2)
Từ (1) và (2) ta có đẳng thức kCn = nCn−1 .
k k−1

2. Xét bài toán "Từ n thành viên của đội văn nghệ, có bao nhiêu cách lập một nhóm
hát trong đó có một nhóm trưởng?". Làm tương tự như câu trên ta sẽ có đẳng
thức cần chứng minh.

Bài toán 3. Cho các số nguyên dương n và k với 0 ≤ k ≤ n. Chứng minh rằng:
Pn
1. C k = Cn+1
m=k m
k+1

Pn−k n
2. m=k
Cmk Cn−m
k
= Cn+1
2k+1
với 0 ≤ k ≤ .
2
Lời giải
1. Xét tập X = 1, 2, ..., n + 1. Khi đó ta đếm số tập con có k + 1 phần tử của X .
k+1
Cách 1: Rõ ràng số tập con là Cn+1 .
Cách 2: Ta chọn tập con sao cho phần tử lớn nhất là m. Khi đó số tập con có
phần tử lớn nhất m là Cmk . Vì k ≤ m ≤ n nên ta có số tập con là Ckk +Ck+1
k
+...+Cnk .
Từ đó suy ra đẳng thức cần chứng minh.

2. Xét bài toán "Đếm số tập con có 2k + 1 phần tử của X ".


Cách 1: Số tập con là Cn2k+1 .
Cách 2: Ta xét phần tử thứ k + 1, giả sử đó là m, khi đó ta chọn k phần tử nhỏ
hơn m và k phần tử lớn hơn m, số cách chọn là Cmk Cn−mk
, vì k ≤ m ≤ n − k nên
Pn−k k k
ta có số cách chọn là m=k Cm Cn−m . Từ đó ta có đẳng thức cần chứng minh.

Tiếp theo là các bài tập áp dụng:

Bài 1 Cho 0 ≤ m ≤ k ≤ n. Chứng minh các đẳng thức sau:


1. Cnk Cmk = Cnm Cn−m
k−m

k≥0 k(Cn ) = nC2n−1


P k 2 n−1
2.

k≥0 Cn Cn−k = 2 Cn
P k m−k m m
3.

Bài 2. Chứng minh các đẳng thức sau:


1. Cn0 .Cnk + Cn1 .Cn−1
k−1
+ ... + Cnk .Cn−k
0
= 2k .Cnk
m
2. k.Cmk .C p0 + (k − 1).Cmk−1 .C p1 + ... + Cm1 C pk−1 = k
.k.Cm+p
m+p
3. Cnm .Ck0 + Cn−1
m−1 1
.Ck+1 + ... + Cn−m
0 m
.Ck+m = Cn+k+1
m

Tập san Toán học STAR EDUCATION


NGUYỄN TĂNG VŨ

3. Các bài toán chứng minh

Bài toán 4. Trong một hội nghị, mỗi thành viên tham gia đúng 3 cuộc họp và mỗi
cuộc họp thì có đúng 6 thành viên tham gia. Chứng minh rằng số cuộc họp thì bằng
nửa số thành viên tham gia hội nghị.

Lời giải Gọi số thành viên là n, số cuộc hộp là m. Khi đó mỗi cuộc họp có 6 thành
viên tham gia, nên tổng số lượt thành viên tham gia m cuộc họp là 6m (có lặp lại).
Tương tự mỗi thành viên tham gia 3 cuộc họp mà có n thành viên nên số lượt thành
viên tham gia là 3n. Do đó 3n = 6m hay n = 2m.
Trong bài toán trên ta có thể làm như sau: giả sử có m cuộc họp là 1, 2, ..., m và n
thành viên là 1, 2, 3, ..., n. Xét bảng vuông m × n gồm m dòng và n cột trên đó ghi các
số dòng thứ i cột j là ai j thỏa ai j = 1 nếu người j tham gia cuộc họp thứ i và ai j = 0
trong trường hợp ngược lại. Ta được bảng sau:

Dựa vào trên, ta thấy mỗi dòng có 6 số 1 và mỗi cột có 3 số 1. Khi đó ta có 6m = 3n


hay n = 2m.
Bảng trên được gọi là một ma trận nhị phân, dùng để biểu diễn các mối quan hệ hai
ngôi như phần tử thuộc tập hợp, quen nhau, đồ thị... và là mô hình biểu diễn rất hữu
dụng trong các bài toán tổ hợp. Trong mỗi bảng nhị phân trên, nếu gọi ri là số số 1 ở
dòng thứ i và c j là số số 1 ở cột thứ j, ta có
m
X n
X
ri = cj.
i=1 j=1

Bài toán 5. (Hồng Kông 1994) Trong một trường học có m giáo viên và n học sinh
thỏa điều kiện sau:

i) Mỗi giáo viên dạy đúng p học sinh.

ii) Với hai học sinh phân biệt thì có đúng q giáo viên dạy họ.
m n(n − 1)
Chứng minh rằng =
q p(p − 1)

Lời giải
Lập bảng gồm m dòng và n cột trong đó ai j = 1 nếu giáo viên i dạy học sinh j, và
bằng 0 nếu ngược lại. Khi đó từ (i) thì mỗi dòng có đúng p số 1. Ta đếm các cặp số
(1; 1) trên cùng một dòng. Nếu đếm theo dòng thì mỗi dòng có C p2 cặp, có m dòng
nên số cặp là mC p2 . (1)
Nếu đếm theo cột, do điều kiện (ii) nên với hai cột bất kì thì có đúng q cặp. Do đó số

Tập san Toán học STAR EDUCATION


58 NGUYỄN TĂNG VŨ

m n(n − 1)
cặp là qCn2 (2). Từ (1) và (2) ta có mC p2 = qCn2 hay= .
q p(p − 1)
Trên đây là một kĩ thuật đếm theo cặp (1; 1) cùng một dòng hoặc cùng một cột. Ta có
mệnh đề sau:

Định lý 1. Nếu trong một bảng nhị phân m × n, mỗi dòng có k số 1, hai cột bất kỳ có
đúng p cặp (1; 1) cùng một dòng. Khi đó ta có pCn2 = kCm2 .

Tiếp theo là các bài tập áp dụng:

1. Cho tập X = 1, 2, ..., 8 và các tập A1 , A2 , ..., A6 là các tập con của X sao cho mỗi
tập Ai có 4 phần tử và mỗi phần tử của S thuộc m tập Ai . Tìm m.

2. Trong một vòng thi toán chung kết tại trường PNTK, các thí sinh phải giải 9 bài
toán. Biết rằng mỗi thí sinh giải được đúng 6 bài, và với hai thí sinh bất kì thì
giải đúng chung 3 bài. Tìm số thí sinh dự thi.

3. Gọi p(n, k) là số hoán vị của 1, 2, ..., n có k điểm bất động. Chứng minh rằng:
n
X
kp(n, k) = n!.
k=1

4. Các bài toán về bất đẳng thức và cực trị tổ hợp

Bài toán 6. (Iran 2011) Cho n điểm trên mặt phẳng sao cho không có 3 điểm nào
thẳng hàng. Chứng minh rằng số tam giác có diện tích bằng 1 có các đỉnh thuộc n
2
điểm trên không vượt quá (n2 − n).
3

Lời giảiBài toán này ta đi tính số cặp (cạnh;tam giác). Với đoạn thẳng AB, khi đó nếu
2
điểm C thỏa SABC = 1 thì khoảng cách từ C đến AB bằng , vì không có 3 điểm nào
AB
thẳng hàng nên chỉ có nhiều nhất 4 điểm thỏa. Vậy với 1 đoạn ta sẽ có nhiều nhất 4
tam giác có diện tích 1 nhận đoạn thẳng đó làm đỉnh. Suy ra tổng số cặp nhiều nhất
là 4Cn2 .
Mặt khác nếu gọi số tam giác là m thì tổng số cặp là 3m. Từ đó ta có:

2
3m ≤ 4Cn2 hay m ≤ (n2 − n).
3

Bài toán 7. (IMO 1998, P2) Trong một cuộc thi có a thí sinh và b giám khảo, với
b là số lẻ lớn hơn 3. Mội giám khảo có thể đánh giá thí sinh rớt hay đậu.Giả sử với
hai giám khảo bất kì thì quyết định giống nhau nhiều nhất là k thí sinh. Chứng minh
k b−1
rằng ≥
a 2b

Tập san Toán học STAR EDUCATION


NGUYỄN TĂNG VŨ

Lời giải Cũng như ví dụ trên, ta thấy việc biểu diễn các mối quan hệ bằng bảng nhị
phân rất thuận lợi trong việc trình bày lời giải. Trong bài này ta cũng có thể lập bảng
b × a theo quy tắc sau: dòng i cột j bằng 1 nếu giám khảo i cho thí sinh j đậu. Ta sẽ
đếm số cặp (0; 0) và (1; 1) cùng một cột bằng hai cách.
Cách 1 ta đếm theo dòng: Vì với hai vị giáo bất kì có nhiều nhất k kết luận giống nhau
nên với hai dòng bất kì có k cặp, do đó số cặp nhiều nhất là kC b2 .
Cách 2 ta đếm theo cột: Trong mỗi cột số cặp là Cm2 + Cn2 cặp, trong đó m là số các số
0 và n là số các số 1, ta có m + n = b = 2t + 1, suy ra n = 2t + 1 − m. Khi đó

m(m − 1) + (21 − m)(2t − m − 1) (2t − m)2 + m2 (b − 1)2


Cm2 + Cn2 = = ≥ t2 = .
2 2 4

Từ đó ta có
a(b − 1)2 k b−1
kC b2 ≥ suy ra ≥
4 a 2b.

Bài toán 8. (USA TST 2005) Cho n > 1. Với số nguyên dương m. Đặt X m = {1, 2, ..., mn}.
Xét họ T gồm 2n tập hợp thỏa các điều kiện sau:

i) Mỗi phần tử của T là một tập con có m phần tử của X m .

ii) Mỗi cặp thuộc T có nhiều nhất một phần tử chung.

iii) Mỗi phần tử thuộc X m thuộc đúng hai tập của T.


Tìm giá trị lớn nhất của m theo n.

Lời giải Xét bảng vuông sao cho gồm 2n dòng và mn cột sao cho ai j = 1 nếu số j
thuộc ai và bằng 0 trong trường hợp ngược lại.
Ta xét bài toán đếm số cặp (1; 1) cùng một cột. Do (i) nên ta có số cặp nhiều nhất là
2
C2n . Do (ii) nên ta có số cặp là mn.
Do đó mn ≥ C2n 2
, suy ra m ≥ 2n − 1. Nếu m = 2n − 1, ta xét mô hình sau. Cho 2n
đường thẳng không có 3 đường nào đồng quy và không có hai đường nào song song.
Khi Xm là tập các giao điểm và T là họ gồm các điểm thuộc một đường thẳng. Rõ
ràng đây là mô hình thỏa đề bài. Bảng sau cho ví dụ n = 2, m = 3.

Bài toán 9. Cho n điểm trong


p mặt phẳng. Chứng minh rằng số cặp điểm có khoảng
n 2n3
cách bằng 1 không quá + .
4 2

Tập san Toán học STAR EDUCATION


60 NGUYỄN TĂNG VŨ

Lời giải Gọi di là số đoạn thẳng đơn vị có đỉnh là Ai . Đặt khi đó số cặp điểm là
1
k = (d1 + d2 + ... + dn ).
2
Ta đếm số cặp (A, B) mà khoảng cách từ A, B đến Ai bằng 1. Số cặp là Cdi
2
, suy ra tổng
Pn
số cặp là i=1 Cdi . Ta biết rằng hai điểm C, D thì có chung nhiều nhất một cặp (A, B)
2
Pn
nên số cặp không vượt quá 2Cn2 . Do đó: i=1 Cd2i ≤ n(n − 1) hay

2k(2k − n)
≤ n(n − 1) ⇔ 2k2 − nk − n2 (n − 1) ≤ 0.
2n
p
n 2n3
Do đó k ≤ + .
4 2

5. Các bài toán tồn tại

Bài toán 10. Cho 133 số nguyên dương, có ít nhất 799 cặp số là nguyên tố cùng
nhau. Chứng minh rằng tồn tại 4 số nguyên dương phân biệt a, b, c, d sao cho a và
b; b và c, c và d; d và a nguyên tố cùng nhau.

Lời giải Mỗi số được đại diện bởi một điểm, hai số nào nguyên tố cùng nhau thì
hai điểm tương ứng được nối nhau bởi một đoạn. Ta cần chứng minh có 4 đoạn
AB, BC, C D, DA. Ta cần chứng minh rằng có hai điểm B và D cùng nối với hai điểm A
và C.
Gọi di là số cạnh có đỉnh là Ai . Khi đó ta có d1 + d2 + ... + d133 = 2. Nếu hai đỉnh Y, Z
cùng nối với đỉnh X thì ta sẽ xem (Y ; Z) là một cặp. Ta sẽ tính số cặp này. Rõ ràng,
tổng số cặp là
133
‚ 133 133
Œ
X 1 X X
Cd2i = di2 − di
i=1
2 i=1 i=1
P133 1 €P133 Š2
Theo BĐT Cauchy-Schwarz thì i=1 di2 ≥ i=1 i
d nên do đó
133
 
133
‚ 133 Œ2 133
X 1 1 X X
Cd2i ≥  di − di  > C133
2
.
i=1
2 133 i=1 i=1

2
Nhưng 133 điểm thì có C133 cặp, nên sẽ có một cặp nào đó được tính hai lần, tức là
tồn tại cặp (A, C) cùng được nối với B và D; tức là ta có 4 đoạn AB, BC, C D, DA.

Bài toán 11. Cho tập X có n phần tử, gọi A1 , A2 , ..., Am là một họ các tập con của X ,
sao cho |Ai | = 3 và |Ai ∩ A j | ≤ 1 với i 6= j. Chứng minh rằng tồn tại một tập con A của
p
X có ít nhất [ 2n] phần tử và không chứa bất kì tập Ai nào.

Lời giải Ta xét tập tất cả các tập con của X mà không chứa bất kỳ tập Ai nào, khi
đó dễ thấy tập này là khác rỗng (xét tập có 2 phần tử là thỏa) và hữu hạn, nên

Tập san Toán học STAR EDUCATION


NGUYỄN TĂNG VŨ

tồn tại một tập M có nhiều phần tử nhất. Đặt |M | = k. Khi đó, do M có số phần
tử lớn nhất nên mọi tập có số phần tử lớn hơn M đều chứa một tập Ai . Xét tập
M 0 = X \ M = {a1 , a2 , ..., an−k }. Khi đó M ∪ {ai } có k + 1 phần tử, nên theo cách xác
định M thì sẽ tồn tại Ai ⊂ M 0 , do Ai * M nên Ai = {ai , x, y} trong đó x, y ∈ M .
Hơn nữa hai tập giao nhau có không quá một phần tử nên với mỗi ai có nhiều nhất
một cặp (x, y) ∈ Ai . Ta đếm số cặp (x, y) theo hai cách:

• Số cặp (x, y) ∈ X là Ck2 .

• Vì i = 1, 2, ..., n − k nên có n − k cặp nên ta có: n − k ≤ Ck2 ⇔ k2 + k ≥ 2n

p p
Mà k ≤ k2 + k ≤ k + 1, suy ra k ≥ [ 2n]. Ta có điều cần chứng minh.
Cuối cùng là một số bài toán rèn luyện:

1. Cho 7 tập A1 , A2 , ..., A7 là các tập con của X = {1, 2, 3, 4, 5, 6, 7}, sao cho mội cặp
phần tử thuộc X thuộc đúng một tập con, và |Ai| ≥ 3 với mọi i. Chứng minh
rằng |Ai ∩ Aj| = 1 với mọi i, j.

2. Cho 16 bạn học sinh làm một bài kiểm tra trắc nghiệm, trong đó mỗi câu hỏi có
4 lựa chọn. Sau bài kiểm tra, ta thấy rằng với hai học sinh bất kì có nhiều nhất
một câu trả lời giống nhau. Hỏi bài kiểm tra có nhiều nhất bao nhiêu câu hỏi?

3. Một hội nghị có n thành viên tham gia, hội nghị đã tổ chứng n + 1 cuộc họp,
trong đó mỗi cuộc họp có đúng 3 người và không có cuộc họp nào có thành viên
giống nhau. Chứng minh rằng có hai cuộc họp mà có chung đúng một thành
viên.

4. (China 1996) Trong một hội nghị có 8 người tham gia, hội nghị tổ chức m cuộc
họp, mỗi cuộc họp có đúng 4 người tham gia. Hơn nữa hai người bất kì thì cùng
tham gia một số cuộc họp như nhau. Tìm giá trị nhỏ nhất của m.

5. Cho A1, A2, ..., Ak là các tập con của S = {1, 2, ..., 10} sao cho:
i) |Ai | = 5, i = 1, 2, ..., k.
ii) |Ai ∩ A j | ≤ 2, 1 ≤ i < j ≤ k. Tìm giá trị lớn nhất của k.

6. (IMO 2001) Có 21 bạn nam và 21 bạn nữ tham dự một kì thi học sinh giỏi toán.
Biết rằng:

i) Mỗi bạn giải được nhiều nhất sáu bài.


ii) Mỗi cặp một nam và một nữ thì có ít nhất một bài toán được giải bởi hai
người đó

Chứng minh rằng có môt bài toán mà giải được bởi ít nhất 3 nam và 3 nữ.

7. (USAMO 2001) Có 8 hộp, mỗi hộp chứa 6 viên bi. Mỗi viên bi được tô màu sao
cho:
i) Mội hộp chứa các viên bi khác màu.
ii) Không có hai màu nào cùng xuất hiện nhiều hơn trong một hộp. Tìm số màu
ít nhất cần dùng.

Tập san Toán học STAR EDUCATION


62 NGUYỄN TĂNG VŨ

8. (IMO 1989) Cho n và k là các số nguyên dương và S là tập n điểm trong mặt
phẳng sao cho:
i) Không có 3 điểm nào thẳng hàng,
ii) Với điểm P bất kì thuộc S thì có ít nhất k điểm của S cách đều P.
1 p
Chứng minh rằng k < + 2n
2
9. (IMO 2005) Trong một cuộc thi toán trong đó đề thi có 6 bài. Mỗi một cặp bài
2
toán được giải bởi nhiều hơn số thí sinh. Không có ai giải được 6 bài. Chứng
5
minh rằng có ít nhất 2 thí sinh giải được đúng 5 bài.

10. Trong một hội nghị có 35 người tham gia. Biết rằng có 110 cặp đôi một quen
nhau. Chứng minh rằng có thể chọn ra 4 thành viên xếp ngồi vào một bàn tròn
sao cho hai người ngồi gần nhau thì quen nhau.

Tập san Toán học STAR EDUCATION


Đề thi tham khảo hướng tới VMO
2019-2020

Kỳ thi chọn HSG quốc gia (viết tắt là VMO) năm nay diễn ra vào các ngày 27, 28
tháng 12/2019. Trong bài viết này, Ban biên tập chúng tôi sẽ đưa ra giới thiệu một đề
thi thử cùng đáp án và phân tích chi tiết. Một số nội dung ở đây đã có giới thiệu tại
group "Hướng tới VMO-TST" trên Facebook.

1. Đề thi

1.1. Đề thi số 1
1. Cho số thực c > 1 và hàm số f : (0; +∞) → R liên tục, thỏa mãn f (x) = f (x c )
với mọi x > 0.
a) Chứng minh rằng f là hàm hằng.
b) Khẳng định trên còn đúng không nếu f không phải là hàm số liên tục?

2. Cho hai đa thức hệ số nguyên, monic là P(x), Q(x), trong đó deg P = 3, deg Q =
2. Giả sử rằng P(x) có ba nghiệm vô tỷ phân biệt có tổng bằng 0 là a, b, c đồng
thời Q(a) = b.
a) Chứng minh rằng P(Q(x)) chia hết cho P(x).
b) Chứng minh rằng Q(a) + Q(b) + Q(c) = 0.

3. Cho tam giác ABC nhọn, không cân nội tiếp trong đường tròn (O) với trực tâm
H. Điểm R thay đổi trên cung lớn BC của (O) sao cho AR không song song với
BC. Lấy các điểm S, T trên đường thẳng BC sao cho (ARS), (ART ) cùng tiếp xúc
với BC. Đường thẳng qua H, vuông góc với AS, AT lần lượt cắt (H BC) ở X , Y.
a) Chứng minh rằng đường thẳng X Y luôn đi qua điểm cố định.
b) Chứng minh rằng tâm của đường tròn (RST ) di chuyển trên đường thẳng cố
định.

4. Tìm tất cả các hàm số f : R → R thỏa mãn điều kiện f (x − 3 f ( y)) = f (x +


f ( y) + y 3 ) + f (4 f ( y) + y 3 ) + 1 với mọi x, y ∈ R.

5. Cho tập hợp S gồm n số square-free lớn hơn 1 có tích bằng m là một số nguyên
dương có đúng 13 ước nguyên tố phân biệt. Biết rằng bất kỳ 5 số nào trong S
cũng không có ước nguyên tố chung và tích 2 số bất kỳ trong S thì không là số
square-free.
a) Chứng minh rằng n ≤ 13.
b) Chứng minh rằng khi n = 13 thì m là số chính phương và mỗi số trong S có
đúng 16 ước nguyên dương.

63
64 BAN BIÊN TẬP

1.2. Đề thi số 2

1. Cho dãy số thực (x n ) thỏa mãn

x 0 = 1, x nn = x n−1
n
+ n với n ≥ 1.

1
Chứng minh rằng với mọi n > 1 thì x n − x n−1 < 2n−1 , từ đó suy ra dãy có giới
hạn hữu hạn không vượt quá 3.

2. Cho tam giác ABC nhọn nội tiếp đường tròn (O). Trên các cạnh AB, AC, lần lượt
lấy các điểm M , N . Đường tròn đường kính BN , C M cắt nhau tại P, Q. Giả sử
rằng P ∈ (O).
a) Chứng minh rằng Q thuộc đường tròn Euler của tam giác ABC.
b) Chứng minh rằng M , N , O thẳng hàng.

3. Tìm tất cả cặp số nguyên dương m, n sao cho 5m + 3m và 5n + 3n cùng chia hết
cho mn.

4. Với các số nguyên dương m, n, xét bảng ô vuông m × n được chia thành các ô
vuông con. Giả sử có thể tô màu trắng – đen bảng này sao cho với mỗi ô vuông
đơn vị thì số ô vuông có cùng màu và có điểm chung với nó (không tính nó) là
số lẻ.
a) Chứng minh rằng mn là số chẵn.
b) Giả sử mn chẵn, chứng minh rằng có thể tô màu thỏa mãn điều kiện đề bài.

5. Hỏi có bao nhiêu cách sắp xếp các số 1, 2, 3, . . . , 10 thành dãy a1 , a2 , . . . , a10 sao
cho ak < a[ k ] với mọi 2 ≤ k ≤ 10?
2

6. Cho tam giác ABC nhọn không cân và đường tròn (O) qua B, C cắt các cạnh
AB, AC ở D, E. Giả sử BE cắt C D ở I. Gọi M , N là trung điểm BE, C D và M N cắt
AB, AC ở P, Q.
a) Chứng minh rằng AI là đường đối trung của tam giác APQ.
b) Chứng minh rằng (APQ) tiếp xúc với đường tròn đường kính IO.

2n 2n−2 2
+x +···+x +1
7. a) Với n ∈ Z+ và x > 0, chứng minh rằng xx 2n−1 n+1
+x 2n−3 +···+x ≥ n .
b) Tìm n nguyên dương nhỏ nhất để tồn tại đa thức P(x) có bậc 2n thỏa mãn
i. Đa thức này có ít nhất một nghiệm thực.
ii. Tất cả các hệ số đều là các số thực thuộc đoạn [19; 20].

8. Tìm tất cả các hàm số f : R → R thỏa mãn

f (|x| + y + f ( y + f ( y))) = 3 y + | f (x)|

với mọi x, y ∈ R.
a) Chứng minh rằng f (x) = 0 ⇒ x = 0 và f là hàm số lẻ.
b) Chứng minh rằng f cộng tính trên R+ , từ đó tìm tất cả các hàm số thỏa mãn
đề bài.

Tập san Toán học STAR EDUCATION


BAN BIÊN TẬP

1.3. Đề thi số 3
1. Tìm tất cả các hàm f : R → R thỏa mãn

f x 2 − f 2 ( y) = x f (x) + y 2 , ∀x, y ∈ R.

2. Cho dãy số (un ) được xác định như sau



u1 = 5,
3n + 2
un+1 = (un + 1), ∀n = 1, 2, 3, . . .
4n + 2

Chứng minh dãy số đã cho có giới hạn hữu hạn và tìm giới hạn đó.

3. Cho tam giác nhọn không cân ABC có các đường cao AD, BE, C F đồng quy tại
trực tâm H. Gọi K, M lần lượt là trung điểm AH và BC. Giả sử BK cắt AM tại L.
a) Chứng minh rằng ∠LE D = 90◦ .
b) Đường thẳng qua tâm ngoại tiếp O của ABC cắt AC ở I. Chứng minh rằng
I K k LE.

4. Tìm tất cả bộ ba các số nguyên tố phân biệt (p, q, r) sao cho

3p4 − 5q4 − 4r 2 = 26.

5. Trong CLB của Nam, các thành viên có thể quen hoặc không quen biết nhau.
Nam nhận thấy rằng: trong mọi khả năng thì chỉ có thể xếp được tối đa 21
thành viên của CLB lên 1 ghế dài sao cho 2 bạn ngồi cạnh thì quen nhau. Hỏi
nếu Nam không ngồi trên ghế đó thì quen được nhiều nhất bao nhiêu bạn ngồi
trên ghế?

6. Trên bàn cờ 8 × 8, bạn A sẽ đặt vào đó hai quân mã đen, còn bạn B sẽ đặt vào
hai quân mã đỏ sao cho không có hai quân mã cùng màu nào có thể ăn nhau
(mỗi quân mã một ô). Gọi a là số bàn cờ khác nhau có thể xảy ra (coi vị trí các
ô trên bàn cờ là phân biệt). Tìm số dư của a khi chia cho 7.

7. Cho các số a, b, c ∈ R đôi một phân biệt sao cho:


• x 2 + ax + 1 = 0, x 2 + bx + c = 0 có nghiệm thực chung.
• x 2 + x + a = 0, x 2 + c x + b = 0 có nghiệm thực chung. Tính tổng T = a + b + c.

8. Cho tam giác ABC nhọn không cân nội tiếp (O), có M là trung điểm BC và điểm
D 6= M thay đổi trên đoạn BC. Đường tròn (ABD) cắt AC ở E, đường tròn (AC D)
cắt AB ở F. Gọi I là tâm ngoại tiếp tam giác AE F và xét đường tròn (ω) qua B, C
tiếp xúc ngoài với (I) ở T. Giả sử rằng BE ∩ C F = K, AK ∩ E F = R, (I) ∩ (O) =
{A, P} và AP ∩ BC = S.
a. Chứng minh rằng RD⊥BC và ST ⊥I T.
b. Chứng minh rằng R, T, M thẳng hàng.

2. Lời giải đề thi số 1

Tập san Toán học STAR EDUCATION


66 BAN BIÊN TẬP

Bài 13. Cho dãy số thực (x n ) thỏa mãn

x 0 = 1, x nn = x n−1
n
+ n với n ≥ 1.
1
Chứng minh rằng với mọi n > 1 thì x n − x n−1 < 2n−1 , từ đó suy ra dãy có giới hạn hữu
hạn không vượt quá 3.

Lời giải. Ta có x 1 = 2 và dễ thấy dãy tăng ngặt. Với n > 1, ta có n = x nn − x n−1 n


=
1 1
(x n − x n−1 )(x nn−1 + · · · + x n−1
n−1
) > n(x n − x n−1 )x n−1
n−1
nên x n − x n−1 < x n−1 ≤ 2n−1 . Từ đó
n−1
suy ra
x n = (x n − x n−1 ) + (x n−1 − x n−2 ) + · · · + (x 2 − x 1 ) + x 1
1 1
< n−1 + · · · + + 2 < 3.
2 2
Dãy tăng và bị chặn trên bởi 3 nên sẽ có giới hạn hữu hạn không vượt quá 3.

Bài 14. Cho tam giác ABC nhọn nội tiếp đường tròn (O). Trên các cạnh AB, AC, lần
lượt lấy các điểm M , N . Đường tròn đường kính BN , C M cắt nhau tại P, Q. Giả sử
rằng P ∈ (O).
a) Chứng minh rằng Q thuộc đường tròn Euler của tam giác ABC.
b) Chứng minh rằng M , N , O thẳng hàng.

Lời giải. a) Gọi BB 0 , C C 0 là đường cao của tam giác ABC và H là trực tâm. Dễ thấy PQ
là trục đẳng phương của (BN ), (C M ) và B 0 ∈ (BN ), C 0 ∈ (C M ) nên H có cùng phương
tích đến hai đường tròn này. Suy ra H ∈ PQ.

Đặt k = H B · H B 0 và xét phép nghịch đảo I Hk . Khi đó: P ↔ Q và (O) ↔ (Euler). Vì


P ∈ (O) nên Q ∈ (Euler).

Tập san Toán học STAR EDUCATION


BAN BIÊN TẬP

b) Kẻ đường kính BB1 , C C1 của (O) thì vì có góc vuông nên dễ thấy B1 , P, N và C1 , P, M
thẳng hàng. Áp dụng định lý Pascal, ta có M N đi qua O.

Bài 15. Tìm tất cả cặp số nguyên dương m, n sao cho 5m + 3m và 5n + 3n cùng chia
hết cho mn.

Lời giải. Trước hết, ta thấy nếu m, n cùng chẵn thì 5m + 3m chia hết cho 4. Trong khi
đó 5m + 3m ≡ 1 + (−1)m ≡ 2(mod4),vô lý nên 1 trong 2 số phải lẻ, giả sử là m.
Nếu m > 1, ta gọi p là ước nguyên tố lẻ nhỏ nhất của m. Dễ thấy (p, 3) = (p, 5) =
1. Khi đó 52m ≡ 32m (modp), 5 p−1 ≡ 3 p−1 ≡ 1(modp) theo Fermat nhỏ. Suy ra h =
gcd(p − 1, 2m) cũng thỏa mãn 5h ≡ 3h (modp). Để ý rằng vì p là ước lẻ nhỏ nhất của
m nên gcd(p − 1, m) = 1. Suy ra h = 2, kéo theo 52 ≡ 32 (mod3) → p|16, vô lý.
Do đó, số p không tồn tại và kéo theo m = 1. Thay vào, ta có 8 chia hết cho n. Thử
trực tiếp, ta thấy n = 1, 2 thỏa mãn.
Vậy có các cặp số là (m, n) = (1, 1), (1, 2), (2, 1).

Bài 16. Với các số nguyên dương m, n, xét bảng ô vuông m × n được chia thành các
ô vuông con. Giả sử có thể tô màu trắng – đen bảng này sao cho với mỗi ô vuông đơn
vị thì số ô vuông có cùng màu và có điểm chung với nó (không tính nó) là số lẻ.
a) Chứng minh rằng mn là số chẵn.
b) Giả sử mn chẵn, chứng minh rằng có thể tô màu bảng thỏa mãn điều kiện đề bài.

Lời giải. Điều kiện cần: ta gọi ô kề cạnh/kề đỉnh với một ô vuông cho trước là láng
giềng của nó. Giả sử bảng m × n thỏa mãn điều kiện đề bài.
Ta xét graph G = (V, E) với V là tất cả các ô được tô đen và hai đỉnh được nối với
nhau nếu chúng là láng giềng của nhau. Theo giả thiết thì mỗi đỉnh của graph này
đều có bậc lẻ, chứng tỏ số đỉnh phải chẵn (theo bổ đề bắt tay). Do đó, số ô được tô
đen là chẵn.
Một cách tương tự thì số ô được tô trắng cũng là chẵn nên tổng số ô của bảng là chẵn,
kéo theo mn chẵn.
Điều kiện đủ: Với mn chẵn, giả sử 2|m thì ta tô màu toàn bộ hai hàng đầu tiên là đen,
hai hàng tiếp theo là trắng và cứ thế. Khi đó, mỗi ô có đúng 3 láng giềng cùng màu.
Vì thế nên điều kiện cần tìm là mn là số chẵn.

Bài 17. Hỏi có bao nhiêu cách sắp xếp các số 1, 2, 3, . . . , 10 thành dãy a1 , a2 , . . . , a10
sao cho ak < a[ k ] với mọi 2 ≤ k ≤ 10?
2

Lời giải. Xét sơ đồ hình cây như trên, trong đó mỗi số ở nút trên sẽ tương ứng lớn
hơn số ở nút dưới.
Rõ ràng a1 phải lớn nhất trong tất cả các số nên a1 = 10. Hai nhánh hai bên độc lập
với nhau nên số cách chọn 3 số cho nhánh phải là C93 , tiếp theo a3 lớn nhất trong 3
số đó nên có 1 cách chọn, a6 , a7 tương ứng có 2 cách chọn. Suy ra có 1 × 2 cách để
xếp 3 số đó. Tiếp theo, ta sẽ có a2 lớn nhất trong 6 số ở nhánh trái. Tương tự ta có
C53 × 1 × 2 × 1 × 1 cách sắp xếp các số ở bên dưới.

Tập san Toán học STAR EDUCATION


68 BAN BIÊN TẬP

Từ đó, theo nguyên lý nhân, số cây như trên, cũng chính là số dãy cần tìm là C93 ×
C53 × 22 .

Bài 18. Cho tam giác ABC nhọn không cân và đường tròn (O) qua B, C cắt các cạnh
AB, AC ở D, E. Giả sử BE cắt C D ở I. Gọi M , N là trung điểm BE, C D và M N cắt
AB, AC ở P, Q.
a) Chứng minh rằng AI là đường đối trung của tam giác APQ.
b) Chứng minh rằng (APQ) tiếp xúc với đường tròn đường kính IO.

Lời giải. a) Giả sử DE ∩ BC = F và M N ∩ AF = T. Kéo dài BE, C D cắt AF ở X , Y thì từ


các hàng điểm điều hòa, theo hệ thức Maclaurin, ta có I M · I X = I B · I E = I C · I D =
IN · IY.

Do đó M N Y X nội tiếp. Lại theo hệ thức Newton thì T A2 = T X · T Y = T M · T N


nên TA tiếp xúc với (AM N ). Chú ý rằng AM , AN đẳng giác trong góc A nên theo bổ
đề quen thuộc về đẳng giác, ta cũng có TA tiếp xúc với (APQ). Cuối cùng, vì chùm
A(T I, BC) = −1, mà AT là tiếp tuyến nên AI là đối trung của APQ.
b) Theo định lý Brocard cho tứ giác toàn phần BC E D.AF thì I là trực tâm tam giác
OAF. Giả sử AI ∩ (APQ) = S thì dễ thấy T S cũng là tiếp tuyến của (APQ) do tứ giác
APSQ điều hòa. Do đó T S = TA = T F. Suy ra ∠ASF = 90◦ hay AS⊥F S. Mà AI⊥F O
nên S ∈ F O.

Tập san Toán học STAR EDUCATION


BAN BIÊN TẬP

Dễ thấy rằng S là chân đường cao trong tam giác AF O nên T S tiếp xúc với (IO) ở S.
Từ đó suy ra (APQ), (IO) tiếp xúc nhau tại S.

2n 2n−2 2
+x +···+x +1
Bài 19. a) Với n ∈ Z+ và x > 0, chứng minh rằng xx 2n−1 n+1
+x 2n−3 +···+x ≥ n .
b) Tìm số nguyên dương n nhỏ nhất sao cho tồn tại đa thức P(x) có bậc 2n thỏa mãn
i. Đa thức này có ít nhất một nghiệm thực.
ii. Tất cả các hệ số đều là các số thực thuộc đoạn [19; 20].

Lời giải. a) Nếu x = 1, ta có đẳng thức đúng. Nếu x ∈ (0; 1) thì thay y = 1x > 1, bất
đẳng thức cũng không đổi.
2n+2
Vì thế nên ta có thể giả sử x > 1. Biến đổi được x 2n + · · · + x 2 + 1 = x x 2 −1−1 và
2n+1 2n+2
−1
x 2n−1 +· · ·+x = x x 2 −1−x . Ta đưa về xx 2n+1 −x ≥ n+1
n hay nx
2n+2
−(n+1)x 2n+1 +(n+1)x −n ≥
0. Coi vế trái là hàm số f (x) với x > 1. Ta có

f 0 (x) = n(2n + 2)x 2n+1 − (n + 1)(2n + 1)x 2n + (n + 1)


 
= (n + 1) 2nx 2n+1 − (2n + 1)x 2n+1 + 1 ≥ 0

Bất đẳng thức cuối đúng theo AM − GM cho 2n + 1 số. Vì thế nên f đồng biến và
f (x) > f (1) = 0. Ta có đpcm.
b) Vì các hệ số của đa thức P(x) đều dương nên nghiệm của nó phải âm. Đặt là −x 0

P(x) = a2n x 2n + a2n−1 x 2n−1 + · · · + a2 x 2 + a1 x + a0 .
Suy ra a2n x 02n + · · · + a2 x 02 + a0 = a2n−1 x 02n−1 + · · · + a1 x 0 . Dễ thấy V T ≥ 19(x 02n + · · · +
1), V P ≤ 20(x 02n−1 + · · · + x 0 ) nên

20 x 02n + · · · + 1 n+1
≥ 2n−1 ≥ hay n ≥ 19.
19 x0 + · · · + x0 n

Ta cũng có đa thức P(x) = 19x 38 + 20x 37 + 19x 36 + 20x 35 + · · · + 20x + 19 có nghiệm


x = −1 thỏa mãn đề bài. Vậy nmin = 19.

Bài 20. Tìm tất cả các hàm số f : R → R thỏa mãn f (|x| + y + f ( y + f ( y))) =
3 y + | f (x)| với mọi x, y ∈ R.
a) Chứng minh rằng f (x) = 0 ⇒ x = 0 và f là hàm số lẻ.
b) Chứng minh rằng f cộng tính trên R+ , từ đó tìm tất cả các hàm số thỏa mãn.

Lời giải. a) Dễ thấy f toàn ánh. Giả sử f (a) = 0 và thay x = 0, y = a, ta có

0 = 3a + | f (0)| .

Suy ra a tồn tại duy nhất và a = − 13 | f (0)| ≤ 0. Lại thay x = y = a, ta có f (0) = 3a ≤


0. Lại thay x = −a, y = a thì chú ý rằng |−a| + a = 0, ta có f (0) = 3a + | f (−a)| nên
f (−a) = 0, điều này kéo theo a = −a hay a = 0 (do tính duy nhất ở trên).

Tập san Toán học STAR EDUCATION


70 BAN BIÊN TẬP

f (x)
€ y = 0 thì€f (|x|) = | €f (x)| nên
Thay ŠŠŠ f (x) ≥ 0, ∀x ≥ 0. Xét x > 0 và y = − 3 , ta có
f (x) f (x) f (x)
f x− 3 +f − 3 +f − 3 = 0 nên

f (x) f (x) f (x)


  ‹‹
− +f − +f − = −x
3 3 3
với mọi x > € x =ŠŠŠ
€ 0. Trong €đề bài, thay 0 thì f ( y + f ( y + f ( y))) = 3 y. Thay y →
f (x) f (x) f (x) f (x)
− 3 thì f − 3 + f − 3 + f − 3 = − f (x). So sánh hai đẳng thức trên, ta
có f (−x) = − f (x), ∀x > 0 nên f là€ hàm số €lẻ.
f (x) f (x) f (x)
€ ŠŠŠ
b) Từ tính chất hàm số lẻ, ta có f 3 + f 3 + f 3 = f (x) với mọi x > 0.
f ( y)
Trong đề bài, xét x ≥ 0 và y → 3 , ta có
f ( y) f ( y) f ( y)
   ‹‹‹
f x+ +f +f = f ( y) + f (x)
3 3 3
hay f (x + y) = f (x) + f ( y) với mọi x, y > 0. Vì f cộng tính trên R+ nên ta có
f (x) = ax, ∀x > 0. Lại do tính chất hàm lẻ, ta suy ra f (x) = ax, ∀x ∈ R. Thay vào
đề bài, ta có a = 1. Vậy tất cả các hàm số cần tìm là f (x) = x.

3. Đề thi số 2

Bài 1. Tìm tất cả các hàm f : R → R thỏa mãn



f x 2 − f 2 ( y) = x f (x) + y 2 , ∀x, y ∈ R.

Lời giải. Thay x = y = 0, ta có f (− f 2 (0)) = 0 nên tồn tại u =


 − f (0) để f (u) = 0.
2

u=0
Thay x = 0, y = u, ta có f (0) = u2 nên u = − f 2 (0) = −u4 hay .
u = −1
Nếu u = −1 thì f (−1) = 0, f (0) = 1. Thay x = −1, y = 0, ta có f (0) = − f (−1) nên
u = 0, vô lý. Suy ra f (0) = 0. Thay y = 0, ta có f (x 2 ) = x f (x), lại thay x → −x, dễ
dàng có f là hàm lẻ. Mặt khác, thay x = 0 thì f (− f 2 ( y)) = y 2 nên f toàn ánh trên
R+ , kết hợp với hàm lẻ, ta thấy f toàn ánh trên R. Ta viết giả thiết lại thành
f (x 2 − f 2 ( y)) = f (x 2 ) + f (− f 2 ( y)).
Thay x 2 , f 2 ( y) → x, y ≥ 0 ta thấy rằng f (x − y) = f (x) + f (− y) với mọi x, y ≥
0. Kết hợp với yếu tố hàm lẻ, dễ dàng có được f cộng tính trên R. Đến đây xét
f (x + 1)2 = (x + 1) f (x + 1) = (x + 1)( f (x) + 1) và f (x + 1)2 = f (x 2 + 2x + 1) =


f (x 2 ) + f (2x) + f (1) = x f (x) + 2 f (x) + f (1). So sánh hai đẳng thức trên, dễ dàng
có được f (x) = ax với a = f (1). Thay vào thử lại, ta được a = −1 nên hàm số cần
tìm là f (x) = −x.


u1 = 5,
Bài 2. Cho dãy số (un ) được xác định như sau 3n + 2
un+1 = (un + 1), ∀n = 1, 2, 3, . . .
4n + 2

Tập san Toán học STAR EDUCATION


BAN BIÊN TẬP

Chứng minh dãy số đã cho có giới hạn hữu hạn và tìm giới hạn đó.

Lời giải. Ta có hai cách xử lý như sau:


Cách 1. Tính toán trực tiếp, ta dự đoán dãy giảm. Xét hiệu

(3n + 2)(un + 1) − (4n + 2)un (3n + 2) − nun


un+1 − un = =
4n + 2 4n + 2

nên ta sẽ đưa về chứng minh rằng un ≥ 3 + 2n với mọi n ≥ 1. Quy nạp, với n = 1 thì
u1 = 5, đúng. Giả sử rằng đã có un ≥ 3 + 2n thì

3n + 2 2 3n + 2 2 2
 ‹
un+1 ≥ 4+ = =3+ >3+ .
4n + 2 n n n n+1

Từ đó suy ra dãy giảm, bị chặn dưới bởi 3 nên hội tụ. Đặt lim un = L ≥ 3 thì L =
3
4 (L + 1) ⇒ L = 3.
3n+2 2
Cách 2. Dự đoán lim un = 3 nên ta viết un+1 − 3 = 4n+2 (un − 3) + 4n+2 nên

3n + 2 2 5 2
|un+1 − 2| ≤ |un − 3| + ≤ |un − 3| + .
4n + 2 4n + 2 6 4n + 2

Áp dụng bổ đề: Cho hai dãy dương (an ), (bn ) với lim bn = 0 và an+1 ≤ qan + bn với mọi
n. Khi đó ta cũng có lim an = 0. Từ bổ đề suy ra lim |un − 3| = 0 hay lim un = 3.

Bài 3. Cho tam giác nhọn không cân ABC có các đường cao AD, BE, C F đồng quy tại
trực tâm H. Gọi K, M lần lượt là trung điểm AH và BC. Giả sử BK cắt AM tại L.
a) Chứng minh rằng ∠LE D = 90◦ .
b) Đường thẳng qua tâm ngoại tiếp O của ABC cắt AC ở I. Chứng minh rằng I K k LE.

Lời giải. Để có ∠LE D = 90◦ , ta cần có ∠LEA = ∠DEB = ∠F EB. Gọi N là trung điểm
E F. Ta thấy rằng hai tia AN , AM đẳng giác trong góc A nên hai tia AN , AL đẳng giác
trong tam giác ABE.

Tập san Toán học STAR EDUCATION


72 BAN BIÊN TẬP

Lại có ABH, EBF đồng dạng nghịch nên BK, BN là hai trung tuyến tương ứng sẽ đẳng
giác với nhau trong góc B. Từ đó ta được L, N là hai điểm liên hợp đẳng giác trong
tam giác ABE.Điều này kéo theo ∠LEA = ∠N EB.
b) Theo câu a thì ta chỉ cần chứng minh ∠BK I = 90◦ là được. Kẻ I J⊥BC với J ∈ BC.
Dễ thấy rằng I J = OM = H K = KA nên có các tứ giác H K I J, AK J I là hình bình hành.
Suy ra K J k AC nên BH⊥K J, chứng tỏ H là trực tâm của tam giác BK J. Do đó,
H J⊥BK, mà H J k K I nên ta được BK⊥K I. Ta có đpcm.

Bài 4. Tìm tất cả bộ ba các số nguyên tố phân biệt (p, q, r) sao cho

3p4 − 5q4 − 4r 2 = 26.

Lời giải. Xét modulo 3, nếu q, r > 3 thì 3p4 − 5q4 − 4r 2 ≡ 0 (mod3), mâu thuẫn. Do
đó q = 3 hoặc r = 3. Ta xét hai trường hợp:
(1) Nếu q = 3, ta có 3p4 − 4r 2 = 431, xét mod5 thì V T ≡ 3 + r 2 ≡ V P ≡ 1 nên
r 2 ≡ −2 (mod5), vô lý.
(2) Nếu r = 3 thì 3p4 − 5q4 = 62. Nếu p > 5 thì V T ≡ 3 ≡ V P ≡ 2 (mod5), vô lý.
Do đó, ta có p = 5, và tìm được r = 19.

Bài 5. Trong CLB của Nam, các thành viên có thể quen hoặc không quen biết nhau.
Nam nhận thấy rằng: trong mọi khả năng thì chỉ có thể xếp được tối đa 21 thành viên
của CLB lên 1 ghế dài sao cho 2 bạn ngồi cạnh thì quen nhau. Hỏi nếu Nam không
ngồi trên ghế đó thì quen được nhiều nhất bao nhiêu bạn ngồi trên ghế?

Lời giải. Nam không thể quen hai bạn ngồi hai đầu ghế vì nếu không thì có thể xếp
Nam ngồi lên dãy ghế, mâu thuẫn với tính lớn nhất. Đánh số các thành viên còn lại
ngồi trên ghế là 1, 2, . . . , 19.

Khi đó, Nam quen được tối đa 10 bạn có số thứ tự lẻ, vì nếu quen nhiều hơn, Nam sẽ
quen 2 bạn có số thứ tự liên tiếp. Khi đó, Nam có thể ngồi chen giữa vào hai bạn ấy
và cũng mâu thuẫn. Vậy số người Nam có thể quen nhiều nhất là 10.

Bài 6. Trên bàn cờ 8 × 8, bạn A sẽ đặt vào đó hai quân mã đen, còn bạn B sẽ đặt vào
hai quân mã đỏ sao cho không có hai quân mã cùng màu nào có thể ăn nhau (mỗi
quân mã một ô). Gọi a là số bàn cờ khác nhau có thể xảy ra (coi vị trí các ô trên bàn
cờ là phân biệt). Tìm số dư của a khi chia cho 7.

Tập san Toán học STAR EDUCATION


BAN BIÊN TẬP

Lời giải. Xét vị trí quân mã đen – đỏ như các dấu X và dấu O. Gọi S là tập hợp các
vị trí có thể có của 2 dấu X và 2 dấu O. Hai ô ở vị trí (a, b) và (c, d) nào đó mà
|(a − c)(b − d)| = 2 được gọi là “liên kết” nhau. Gọi SX ⊂ S là các vị trí mà 2 dấu X
liên kết nhau. Định nghĩa tương tự với SO , SX O . Với mỗi ô trong bảng 8 × 8, ta viết vào
đó số lượng các ô liên kết với nó như bên dưới:

Tổng số lượng các số trên bảng cũng chính bằng hai lần số cặp ô liên kết nhau (theo
bổ đề bắt tay) và bằng
4 × 2 + 8 × 3 + 20 × 4 + 16 × 6 + 16 × 8
= 168.
2
Từ đó, ta dễ dàng tính được |S| = C64 2 2
· C62 ≡ 0 (mod7), |SX | = |SO | = 168 · C62
2

0 (mod7). Mấu chốt vấn đề là tính |SX O | . Tuy nhiên, khi chọn một cặp ô, ta vẫn phải
loại trường hợp hai cặp đó có chung một ô. Ta lại tiếp tục đếm số bộ ba (A, {B, C}) mà
(A, B) và (A, C) liên kết nhau. Tổng số bộ là

S 0 X O = 4 × C22 + 8 × C32 + 20 × C42 + 16 × C62 + 16 × C82 ≡ 1 (mod 7).

Do đó |SX O | = C168 2
− |S 0 X O | ≡ 6 (mod7). Theo nguyên lý bù trừ, ta tính được a =
|S| − (|SX | + |SO | − |SX O |) chia 7 dư 6.

Bài 7. Cho các số a, b, c ∈ R đôi một phân biệt sao cho:


• x 2 + ax + 1 = 0, x 2 + bx + c = 0 có nghiệm thực chung.
• x 2 + x + a = 0, x 2 + c x + b = 0 có nghiệm thực chung. Tính tổng T = a + b + c.

Lời giải. Gọi x 1 , x 2 lần lượt là nghiệm chung của hai phương trình bậc hai trên. Ta có

x 12 + ax 1 + 1 = x 12 + bx 1 + c = 0
x 22 + x 2 + a = 0, x 22 + c x 2 + b = 0.

Suy ra
(a − b)x 1 + 1 − c = 0 ⇒ (a − b)x 1 = c − 1


(1 − c)x 2 + a − b = 0 ⇒ (c − 1)x 2 = a − b
Do a 6= b nên c 6= 1, nhân hai đẳng thức trên lại, ta suy ra x 1 x 2 = 1. Để ý rằng phương
trình x 2 + ax + 1 = 0 có nghiệm là x 1 nên còn nghiệm khác là x 0 1 thỏa mãn x 1 x 0 1 = 1,
suy ra x 2 = x 0 1 hay x 2 cũng là nghiệm của x 2 + ax + 1 = 0. Gọi x 0 là nghiệm chung
của phương trình x 2 + ax + 1 = x 2 + x + a = 0 thì x 02 + ax 0 + 1 = x 02 + x 0 + a = 0 nên
(a − 1)x 0 + 1 − a = 0 ⇔ (a − 1)(x 0 − 1) = 0. Dễ thấy a = 1 không thỏa nên x 0 = 1,
từ đó có a = −2, b + c = −1 nên T = −3.

Tập san Toán học STAR EDUCATION


74 BAN BIÊN TẬP

Bài 8. Cho tam giác ABC nhọn không cân nội tiếp (O), có M là trung điểm BC và
điểm D 6= M thay đổi trên đoạn BC. Đường tròn (ABD) cắt AC ở E, đường tròn (AC D)
cắt AB ở F. Gọi I là tâm ngoại tiếp tam giác AE F và xét đường tròn (ω) qua B, C tiếp
xúc ngoài với (I) ở T. Giả sử rằng BE ∩ C F = K, AK ∩ E F = R, (I) ∩ (O) = {A, P} và
AP ∩ BC = S.
a. Chứng minh rằng RD⊥BC và ST ⊥I T.
b. Chứng minh rằng R, T, M thẳng hàng.

Lời giải. a) Từ các tứ giác nội tiếp, ta có ∠AF K + ∠AEK = ∠AF C + ∠AEC = ∠ADB +
∠ADC = 180◦ , suy ra AEK F nội tiếp. Áp dụng định lý Brocard cho tứ giác toàn phần
AEK F.BC với chú ý rằng D là điểm Miquel và R là giao điểm hai đường chéo, ta có
RD⊥BC.

Tiếp theo, ta thấy rằng SP ·SA = SB·SC nên S là tâm đẳng phương của (I), (ABC), (ω).
Do đó, S T tiếp xúc với (I) và ST ⊥AT.

b) Gọi L là tiếp điểm khác T của tiếp tuyến kẻ từ S đến (I).Xét đường tròn (I) thì: BC
là đường đối cực của R, đi qua S nên đối cực của S là LT sẽ đi qua R.
Suy ra RT ⊥SI. Để kết thúc bài toán, ta chỉ cần chứng minh rằng RM ⊥SI hay R là trực
tâm của tam giác SI M . Để ý rằng R là trực tâm tam giác BI C nên có DR· DI = DB · DC.
Do đó, ta đưa về chứng minh DB · DC = DM · DS hay (SD, BC) = −1. Theo phương
tích thì BD · BC = BF · BA, C D · C B = C E · CA nên CBDD = CBFE · AC
AB
.
Để ý rằng hai tam giác P BF, P C E đồng dạng nên C E = P C . Cũng có SP B, SCA đồng
BF PB

dạng nên ACPB


= SB
SA ; tương tự thì SP C, SBA đồng dạng nên AB = SA . Chia xuống, ta có
PC SC

P B AB SB
· = .
P C AC SC

Kết hợp lại ta được BD


CD = SB
SC nên (SD, BC) = −1. Đến đây ta có đpcm.

Tập san Toán học STAR EDUCATION


BAN BIÊN TẬP

4. Lời giải đề thi số 3

Bài 1. Cho số thực c > 1 và hàm số f : (0; +∞) → R liên tục, thỏa mãn f (x) =
f (x c ) với mọi x > 0. a) Chứng minh rằng f là hàm hằng. b) Khẳng định trên còn
đúng không nếu f không phải là hàm số liên tục?

Lời giải. a) Theo giả thiết thì với mọi x > 0, ta có f (x) = f (x 1/c ) nên thực hiện phép
n
thế liên tiếp thì f (x) = f (x 1/c ). Chú ý rằng lim c1n = 0 nên theo tính chất hàm liên
 n→+∞
1/c n n
tục, ta có f (x) = lim f (x ) = f lim x 1/c = f (x 0 ) = f (1). Vì thế nên f là
n→+∞ n→+∞
hàm hằng.
b) Khẳng định sẽ không còn đúng nữa. Chẳng hạn ta chọn hàm số f (1) = 1, f (x) =
2, ∀x 6= 1. Rõ ràng ∀x 6= 1 thì ta đều có x c 6= 1 nên ta luôn có f (x) = f (x c ), ∀x > 0.
Ngoài ra, vì không liên tục nên không thể thực hiện phép thế liên tiếp và lấy giới hạn
như trên, giá trị của f (1) độc lập với các giá trị còn lại, thế nên hàm số trên thỏa mãn
đề bài và rõ ràng nó không phải là hàm hằng.

Nhận xét. Câu hỏi tương tự là: Chứng minh rằng hàm số liên tục f : R+ → R thỏa mãn
điều kiện sau là hàm hằng f (x) = f x 2 + 14 , ∀x ∈ R+ . Đặc điểm chung của các bài


này là có thể thực hiện phép thế liên tiếp để từ một giá trị x bất kỳ, ta tạo được dãy hội
tụ về giá trị cụ thể.

Bài 2. Cho hai đa thức hệ số nguyên, monic là P(x), Q(x), trong đó deg P =
3, deg Q = 2. Giả sử rằng P(x) có ba nghiệm vô tỷ phân biệt có tổng bằng 0 là
a, b, c đồng thời Q(a) = b.
a) Chứng minh rằng P(Q(x)) chia hết cho P(x).
b) Chứng minh rằng Q(a) + Q(b) + Q(c) = 0.

Lời giải. a) Ta sẽ chứng minh rằng P(x) là đa thức nguyên bậc nhỏ nhất nhận x = a
là nghiệm.

Chứng minh. Giả sử ngược lại có đa thức nguyên khác hằng f (x) bậc nhỏ hơn 3 mà
f (a) = 0, rõ ràng f (x) không thể là bậc nhất vì a là số vô tỷ. Suy ra f bậc hai. Xét
phép chia đa thức
P(x) = f (x) · g(x) + r(x)
thì dễ thấy r(a) = 0, và deg r < deg f = 2 nên deg r = 0. Suy ra r(x) ≡ 0, tức là P(x)
chia hết cho f (x), nên P(x) = f (x)g(x), với g(x) ∈ Q[x], chứng tỏ g có nghiệm hữu
tỷ, và nghiệm đó lại là của P(x), vô lý.
Tiếp theo, nếu có đa thức nguyên f1 (x) mà f1 (a) = 0 thì xét phép chia f1 (x) =
P(x) · g1 (x) + r1 (x),ta có r1 (a) = 0, mà deg r1 < deg P nên phải có r1 (x) ≡ 0, tức là
f1 (x) chia hết cho P(x).

Trở lại bài toán, rõ ràng P(Q(a)) = P(b) = 0 nên x = a là nghiệm của P(Q(x)), từ các
nhận xét trên, ta phải có P(Q(x)) chia hết cho P(x).
b) Theo trên, ta có P(Q(b)) = P(Q(c)) = 0. Suy ra Q(b), Q(c) ∈ {a, b, c}.

Tập san Toán học STAR EDUCATION


76 BAN BIÊN TẬP

Ta xét các trường hợp:


Nếu Q(b) = b thì đa thức Q(x) − x có bậc hai nhận nghiệm x = b, vô lý vì P(x) là đa
thức bậc nhỏ nhất thỏa mãn điều này.
Nếu Q(b) = a thì đặt Q(x) = x 2 + px + q, ta có hệ
a + ap + q = b
 2

b2 + bp + q = a

Trừ từng vế, ta được (a − b)(a + b + p) = b − a nên a + b + p = −1 → c = p + 1, vô lý


vì c vô tỷ còn p + 1 nguyên. Do đó Q(b) = c. Chứng minh tương tự ta có Q(c) = a nên
Q(a) + Q(b) + Q(c) = a + b + c = 0.
Nhận xét. Bài toán là một ứng dụng thú vị của “đa thức tối tiểu”, là đa thức nguyên
bậc nhỏ nhất nhận một số vô tỷ là nghiệm. Bài toán gốc được lấy từ đề China TST với
nội dung câu hỏi như sau (giả thiết tương tự): Chứng minh rằng p2 − 2p − 4q − 7 là số
chính phương. Ta xử lý tiếp như sau: từ hệ phương trình

a = b + pb + q
 2

b = c 2 + pc + q
c = a2 + pa + q

(*) Trừ từng vế hai phương trình đầu của (∗) , ta có a − b = b2 − c 2 + p(b − c) =
(b−c)(b+c+p) = (b−c)(p−a). Tương tự thì b−c = (c−a)(p−b) và c−a = (a−b)(p−c)
nên nhân tất cả các hệ thức lại thì (p − a)(p − b)(p − c) = 1. Cộng các đẳng thức trong
(∗) lại, ta được a2 + b2 + c 2 + 3q = 0 ⇔ ab + bc + ca = 32 q. Nhân phương trình thứ
1, 2, 3 của (∗) cho b, c, a rồi cộng lại, ta có
3q q
a b + bc + ca = a3 + b3 + c 3 + p(a2 + b2 + c 2 ) ⇔ = 3abc − 3pq ⇔ abc = + pq.
2 2
Thay tất cả vào đẳng thức (p − a)(p − b)(p − c) = 1, ta được
3pq q
p3 − p2 (a + b + c) + p(ab + bc + ca) − abc = 1 ⇔ p3 + − − pq = 1
2 2
p=1


q = −2(p2 + p + 1)

(1) Nếu q = −2(p2 + p + 1) thì T = p2 − 2p + 8(p2 + p + 1) − 7 = (3p + 1)2 là số chính


phương.
a = b + b + q
 2

(2) Nếu p = 1, khi đó abc = 32 q và b = c 2 + c + q nên


c = a2 + a + q

3q X
= ab + bc + ca = (a2 + a + q)(b2 + b + q)
2
= a2 b2 + b2 c 2 + c 2 a2 − 3abc + ab + bc + ca + 2q(a2 + b2 + c 2 ) + 3q2
9q2 9q 3q 3q2
= − + + 2q(−3q) + 3q2 = − − 3q
4 2 2 4
Do đó, ta có q = 0 hoặc q = −6. Rõ ràng q = 0 không thỏa vì khi đó abc = 0 nên phải
có một số bằng 0. Còn nếu q = −6, p = 1 thì T = 16, cũng là số chính phương.

Tập san Toán học STAR EDUCATION


BAN BIÊN TẬP

Bài 3. Cho tam giác ABC nhọn, không cân nội tiếp trong đường tròn (O) với trực
tâm H. Điểm R thay đổi trên cung lớn BC của (O) sao cho AR không song song với
BC. Lấy các điểm S, T trên đường thẳng BC sao cho (ARS), (ART ) cùng tiếp xúc với
BC. Đường thẳng qua H, vuông góc với AS, AT lần lượt cắt (H BC) ở X , Y.
a) Chứng minh rằng đường thẳng X Y luôn đi qua điểm cố định.
b) Chứng minh rằng tâm của đường tròn (RST ) di chuyển trên đường thẳng cố định.

Lời giải. a) Gọi K là giao điểm của AR, BC. Theo phương tích thì K B · KC = KA· KR =
KS 2 = K T 2 nên theo hệ thức Newton thì (BC, RS) = −1. Vì chùm A(ST, BC) = −1 nên
trực giao đỉnh H với chú ý BH⊥AC, C H⊥AB, ta có H(M N , BC) = A(ST, C B) = −1.
Suy ra tứ giác BM C N điều hòa và M N sẽ đi qua giao điểm hai tiếp tuyến của (H BC)
ở B, C. Rõ ràng đó chính là điểm cố định.

b) Bằng biến đổi góc, ta có

∠SRT = ∠SRK + ∠T RK = ∠ASK + ∠AT K = 180◦ − ∠SAT.

Suy ra hai đường tròn (RST ), (AST ) đối xứng nhau qua đường thẳng BC. Khi đó, để
chứng minh tâm của (RST ) thuộc đường thẳng cố định, ta đưa về chứng minh cho
tâm của (AST ). Gọi D là trung điểm BC và E là giao điểm của AD với (AST ) thì theo
hệ thức Newton thì DA · DE = DT · DS = DB 2 = DC 2 nên E là điểm cố định. Do đó,
đường tròn (AST ) đi qua điểm cố định A, E nên tâm của nó sẽ di chuyển trên trung
trực của AE cũng cố định. Ta có đpcm.

Nhận xét. Các điểm R, E ở trên lần lượt chính là điểm Humpty của các tam giác
AS T, ABC. Khai thác tính chất của các điểm này, ta sẽ còn nhiều kết quả thú vị.

Tập san Toán học STAR EDUCATION


78 BAN BIÊN TẬP

Bài 4. Tìm tất cả các hàm số f : R → R thỏa mãn điều kiện

f (x − 3 f ( y)) = f (x + f ( y) + y 3 ) + f (4 f ( y) + y 3 ) + 1

với mọi x, y ∈ R.

Lời giải. Thay x → 3 f ( y) vào phương trình đề cho, ta có f (0) = 2 f (4 f ( y) + y 3 ) + 1,


f (0)−1 f (0)+1
hay f (4 f ( y) + y 3 ) = 2 = a − 1, ∀ y ∈ R trong đó a = 2 . Thay vào phương
trình đề bài, ta có
f (x − 3 f ( y)) = f (x + f ( y) + y 3 ) + a
với mọi x, y ∈ R. Thay y → 0, x → x + f ( y) + y 3 − f (0), ta có f (x + f ( y) + y 3 −
f (0) − 3 f ( y)) = f (x + f ( y) + y 3 ) + a, ∀x, y ∈ R hay tương đương

f (x − 2 f ( y) + y 3 − f (0)) = f (x + f ( y) + y 3 ) + a, ∀x, y ∈ R.

So sánh hai phương trình trên ta suy ra f (x−3 f ( y)) = f (x−2 f ( y)+ y 3 − f (0)), ∀x, y ∈
R. Thay x → x + 3 f ( y), ta suy ra f (x) = f (x + f ( y) + y 3 − f (0)), ∀x, y ∈ R. Nếu
f ( y) + y 3 − f (0) = 0, ∀ y thì thử lại thấy f ( y) = − y 3 + c không là nghiệm của phương
trình. Ngược lại, nếu tồn tại y0 thỏa f ( y0 ) + y03 − f (0) = c 6= 0 thay vào phương trình
trên, ta suy ra f (x) = f (x + c), ∀x ∈ R. Khi đó thay y → y + c vào đẳng thức trên, ta

f (x) = f (x + f ( y) + ( y + c)3 − f (0)) = f (x + f ( y) + y 3 − f (0), ∀x, y ∈ R.

Thay x → x − f ( y) − y 3 + f (0) ta lại có f (x) = f (x + ( y + c)3 − y 3 ), ∀x, y ∈ R.. Nếu


c > 0 thì ( y + c)3 − y 3 đa thức bậc 2 có hệ số bậc cao nhất dương nên nhận giá trị trong
tập [m; +∞) với m là một số thực nào đó. Do đó f (x) = f (x + r), ∀x ∈ [m; +∞).
Lấy x, y ∈ R bất kỳ mà x > y. Khi đó tồn tại q, r ∈ [m; +∞) thỏa mãn q − r = x − y.
Ta có f ( y) = f ( y + q) = f ( y + q − r) = f ( y + x − y) = f (x). Điều này chứng tỏ
f là hàm hằng. Tương tự nếu c < 0 ta cũng suy ra f hằng. Thay vào đề bài, ta có
f (x) = −1, ∀x thỏa mãn.
Vậy tất cả các hàm số cần tìm là f (x) = −1, ∀x ∈ R.

Nhận xét. Bài toán trên đây được phát triển từ bài toán

f (x − f ( y)) = f (x + y 2n ) + f ( f ( y) + y 2n ) + 1,

với mọi x, y ∈ R. Ở trong bài toán này, sau một vài bước xử lí, ta thu được một trường
hợp f là hàm tuần hoàn với chu kì c. Từ đó thay vào phương trình đề cho, ta sẽ có
f (x) = f (x + ( y + c)2n − y 2n ), với mọi x, y ∈ R. Đến đây, do c 6= 0 nên ( y + c)2n − y 2n
là đa thức bậc 2n − 1 theo y, tức sẽ phủ hết các giá trị của tập R. Từ đó ta có f (x) =
f (x + r), với mọi x, r ∈ R.
Điều này chứng tỏ f là hàm hằng. Tuy nhiên, ở đây, đa thức ( y + c)3 − y 3 là đa thức
bậc hai nên không thể phủ hết giá trị của tập R mà thay vào đó là [m, +∞) với m ∈ R
nào đó (nếu c > 0) và (−∞, m] (nếu c < 0). Điều này đòi hỏi cách xử lí khéo léo hơn.
Một trong những cách đó là: lấy x > y ∈ R bất kì, ta chứng minh f (x) = f ( y). Thật
vậy, vì f (x) = f (x + r) với mọi x ∈ R, r ∈ [m, +∞) nên chọn q = m + x − y.

Tập san Toán học STAR EDUCATION


BAN BIÊN TẬP

Bài 5. Cho tập hợp S gồm n số square-free lớn hơn 1 có tích bằng m là một số nguyên
dương có đúng 13 ước nguyên tố phân biệt. Biết rằng bất kỳ 5 số nào trong S cũng
không có ước nguyên tố chung và tích 2 số bất kỳ trong S thì không là số square-free.
a) Chứng minh rằng n ≤ 13. b) Chứng minh rằng khi n = 13 thì m là số chính
phương và mỗi số trong S có đúng 16 ước nguyên dương.

Lời giải. a) Gọi P là tập hợp các ước nguyên tố của m. Theo giả thiết, ta nhận xét:
- Mỗi số nguyên tố thuộc P sẽ là ước của không quá 4 số thuộc S.
- Vì tích của hai số square-free thuộc S không phải là số square-free nên chúng phải
có ước nguyên tố chung.
Từ đó, đếm số bộ ({a, b}, p) mà a, b ∈ S, p ∈ P và p là ước chung của a, b.
• Cách 1. Đếm theo {a, b}, ta có ≥ Cn2 · 1.
• Cách 2. Đếm theo p, ta có ≤ 13 · C42 = 78.
Do đó Cn2 ≤ 78 → n(n − 1) ≤ 156 nên n ≤ 13.
b) Khi n = 13, dễ thấy rằng khi đẳng thức xảy ra mỗi số nguyên tố là ước của đúng
4 số nên m là một lũy thừa bậc 4 của tích các số nguyên tố, nói cách khác, nó là số
chính phương. Ngoài ra, hai số bất kỳ trong S đều phải có ước nguyên tố chung.
Cuối cùng nếu có số a ∈ S nào đó chỉ có ≤ 3 ước nguyên tố thì mỗi ước đó sẽ thuộc
về đúng 3 số nữa. Khi đó, còn lại 13 − 1 − 3 × 3 = 3 số sẽ không có ước chung với a,
mâu thuẫn. Suy ra mỗi số trong S đều phải có ít nhất 4 ước nguyên tố.
Đến đây, đếm số bộ (a, p) với a ∈ S, p ∈ P và p|a, ta thấy rằng nếu đếm theo a thì sẽ
có ≥ 13 · 4 = 52, còn đếm theo p thì sẽ có đúng 13 · 4 = 52 nên đẳng thức phải xảy ra,
tức là mỗi số trong S đều có đúng 4 ước nguyên tố nên số ước đúng bằng 24 = 16.
Nhận xét. Bài toán trên chính là một trường hợp đặc biệt của bài toán tổng quát thú
vị sau: Một CLB có n > 1 học sinh được phân hoạch thành m > 1 nhóm nhỏ thỏa mãn:
mỗi nhóm có số thành viên bằng nhau; hai nhóm tùy ý có đúng 1 thành viên chung; hai
thành viên tùy ý tham gia chung đúng 1 nhóm.
Chứng minh rằng 2(m + n) − 3 là số chính phương.
Trước hết, đếm số bộ (học sinh, học sinh, nhóm) mà hai học sinh cùng tham gia vào
nhóm, ta có
Cn2 = mCk2 ⇔ n(n − 1) = k(k − 1)m.
Xét một học sinh A tùy ý và gọi sA là số nhóm mà A tham gia, giả sử trong đó có nhóm
1. Ta sẽ đếm số bộ (học sinh, nhóm) mà học sinh khác A, nhóm khác 1 và học sinh này
cùng với A thuộc vào nhóm đó. Đếm theo học sinh, có n − k cách; còn đếm theo nhóm, có
k−1 ⇔ sA = k−1 . Do giá trị này cố định nên
(sA − 1)(k − 1) cách. Từ đó suy ra sA − 1 = n−k n−1

các học sinh đều tham gia vào đúng n−1


k−1 nhóm. * Đếm số bộ (nhóm, nhóm, học sinh) mà
hai nhóm có học sinh tham gia chung, ta có Cm2 = nC 2n−1 hay (k − 1)(m − 1) = k(n − k).
k−1
Giải hệ, ta có m = n = k2 − k + 1 nên
2(m + n) − 3 = 4(k2 − k + 1) − 3 = (2k − 1)2
là số chính phương. Khi m > k2 − k + 1, cấu trúc đẹp trên bị “phá vỡ”, ta đưa về mô hình
định lý sunflower: Xét một tập hợp n phần tử và m tập con của nó, mỗi tập có k phần tử
sao cho hai tập bất kỳ đều có đúng một phần tử chung. Khi đó nếu như m > k2 − k + 1
thì sẽ có 1 phần tử thuộc về tất cả các tập con. Kết quả này chứng minh không khó bằng
nguyên lý Dirichlet.

Tập san Toán học STAR EDUCATION

You might also like